Você está na página 1de 123

Question 1 (2.

00 points)
With regard to relationship selling (also called consultative selling) as compared with traditional selling,
which of the following is NOT true?
Student response:
!ercent
Value
Correct
Response
Student
Response
Answer Choices


a. With relationship selling, follow-
up after the sale is likely to be
more thorough and will have a
longer-term focus.


b. With relationship selling, a
company is more likely to take a
team-based approach to dealing
with customers.


c. Planning is a higher priority in
relationship selling than in
traditional selling.
100.0%

d. A person whose moral character
is at the postconventional level
most likely is not truly a
"relationship salesperson.


e. Relationship selling is less
focused on "closing, and more
focused on improving the
customers "bottom line.


Score: 2 / 2

Question 2 (2.00 points)
SoBe nutritional beverages puts its "twin lizard" logo on every bottle of its products and in all of its
promotional messages. It also sponsors Team Lizard, which is made up of skateboarders, mountain bike
riders, and inline skaters. The lizard is used to remind customers of the product and its benefits. The lizard
and the communications in which it is used belong to the _________ element of SoBe's marketing mix.
Student response:
!ercent
Value
Correct
Response
Student
Response
Answer Choices
100.0%

a. Promotion


b. Price


c. Place


d. Procurement


e. Product


Score: 2 / 2

Question 3 (2.00 points)
The owner of a chain of Mexican restaurants wants to know whether his employees interact with customers
in a satisfactory manner. Which of the following research methods is most appropriate for obtaining data to
answer this research question?
Student response:
!ercent
Value
Correct
Response
Student
Response
Answer Choices


a. A focus group consisting of
employees


b. Mail questionnaire sent to
current customers
0.0%

c. Telephone survey of current and
potential customers
No . . . see pp. 140-150.


d. In-home personal interviews
with the most-regular customers



e. Mystery shopper


Score: 0 / 2

Question 4 (2.00 points)
Television commercials frequently employ a _________________ executional style, in which viewers are
shown the expected benefit of the advertised product. This style of message execution is particularly popular
among advertisers of household cleaning products.
Student response:
!ercent
Value
Correct
Response
Student
Response
Answer Choices


a. mood


b. lifestyle


c. humorous
100.0%

d. demonstration


e. slice-of-life


Score: 2 / 2

Question 5 (2.00 points)
All of the following are elements of the marketing plan EXCEPT:
Student response:
!ercent
Value
Correct
Response
Student
Response
Answer Choices


a. a mission statement.


b. the situation analysis.


c. the target market strategy.


d. the marketing mix.
100.0%

e. a portfolio analysis.


Score: 2 / 2

Question 6 (2.00 points)
When Darren McFerrin went to Target to buy a 4-pack of Gillette Fusion razor blade cartridges, he found
something he hadnt seen before: a 4-pack of cartridges that included, in the same package, a 1.7-ounce
bottle of Gillette body wash. The price of the 4-pack with the trial-sized bottle was the same as the price
that Darren was accustomed to paying for a 4-pack of cartridges. The package labeling made it clear that
the extra value was a limited-time offering. Gillettes limited-time 4-pack of blades + 1.7-ounce body wash
package is an example of:
Student response:
!ercent
Value
Correct
Response
Student
Response
Answer Choices


a. cooperative advertising.
100.0%

b. a premium.


c. product placement.


d. a rebate.


e. online sales promotion.


Score: 2 / 2

Question 7 (2.00 points)
___________________ is the development of a specific marketing mix to influence potential customers
overall perception of a brand, product line, or organization in general. It is related to "the place that a
product occupies in consumers minds relative to competitive offerings.
Student response:
!ercent
Value
Correct
Response
Student
Response
Answer Choices


a. Multisegment targeting


b. Segment profiling and analysis


c. Undifferentiated targeting


d. Optimization
100.0%

e. Positioning


Score: 2 / 2

Question 8 (2.00 points)
Montanas Best is a new precooked meal-in-a-bag product made from bison meat. The company is having
difficulty gaining consumer acceptance of its bison products even though bison is high in protein and low in
cholesterol, fat, and calories. Those who have tried the meat have agreed that it is tender and as delicious
as any choice steak, and it is less expensive. Montana's Best should focus on which of the following sales
promotion tools?
Student response:
!ercent
Value
Correct
Response
Student
Response
Answer Choices


a. Sponsorship
100.0%

b. Sampling


c. Trade promotions


d. In-pack couponing


e. A rebate


Score: 2 / 2

Question 9 (2.00 points)
Opinion leaders are:
Student response:
!ercent
Value
Correct
Response
Student
Response
Answer Choices


a. wealthy, well-educated
individuals.


b. experts on all high-involvement
consumer goods.


c. usually the same individuals for
all social classes.
100.0%

d. people who influence others.


e. easy to locate and target.


Score: 2 / 2

Question 10 (2.00 points)
Large specialty stores such as _________________ are often referred to as "category killers because they
can destroy the profit potential of an entire category of merchandise for other retailers.
Student response:
!ercent
Value
Correct
Response
Student
Response
Answer Choices


a. Kohls


b. Kroger
100.0%

c. Lowes


d. Sears


e. Gabriel Brothers


Score: 2 / 2

Question 11 (2.00 points)
This month, Garrett Jarrett is opening a new workout facility and full-service day spa. Garrett knows that the
success of his business (which he has named "The Sweat, Stink & Suffer Health Spa, also known as "The
Triple S) depends largely upon the following three environmental conditions:


--If the current recession does not end soon, clients and potential clients will not have enough disposable
income to pay their membership fees.


--Residents of the community have a long list of workout facilities from which to choose, including the YMCA,
the YWCA, three national chains, seven local independent gyms, and three church gyms. Gaining market
share may be difficult.


--Hell have to quickly learn and abide by the many laws that apply to running a for-profit service
establishment, such as worker and customer safety regulations and federal, state, and local tax
requirements.


Of the six external environmental factors that are presented in your text, which three appear to have the
greatest impact on Garretts business? That is, which three are illustrated by the above issues?
Student response:
!ercent
Value
Correct
Response
Student
Response
Answer Choices


a. Demographic, competitive, and
economic


b. Political/legal, technological, and
demographic


c. Competitive, social, and
technological


d. Social, political/legal, and
demographic
100.0%

e. Economic, competitive, and
political/legal.


Score: 2 / 2

Question 12 (2.00 points)
The authors of your text define "__________ as "the managerial process of creating and maintaining a fit
between the organizations objectives and resources and the evolving market opportunities.
Student response:
!ercent
Value
Correct
Response
Student
Response
Answer Choices


a. marketing


b. relationship marketing


c. environmental scanning


d. MOA
100.0%

e. strategic planning


Score: 2 / 2

Question 13 (2.00 points)
Mountain Express is a company that provides grocery delivery service to several small communities in Idaho.
The company gives a 2 percent discount on all food customers who have used the service 10 or more times.
An additional 15 orders (for a cumulative total of 25 orders) will boost a customers discount to 4 percent;
an additional 15 (for a cumulative total of 40) orders raises it to 6 percent, and an additional 15 orders (55
cumulative) bumps the discount to 8 percent, which is the maximum discount allowed to any customer. The
relationship marketing strategy that Mountain Express is using is based on:
Student response:
!ercent
Value
Correct
Response
Student
Response
Answer Choices


a. following service delivery
paradigms.


b. building social bonds.
100.0%

c. building financial bonds.


d. opening personal communication
channels.


e. creating structural bonds.


Score: 2 / 2

Question 14 (2.00 points)
Companies that are MOST likely to succeed in the introduction and development of new products typically:
Student response:
!ercent
Value
Correct
Response
Student
Response
Answer Choices


a. establish an environment
conducive to achieving new-
product objectives.


b. make the long-term
commitment needed to support
innovation and new-product
development.


c. capitalize on experience to
achieve and maintain
competitive advantage.


d. use a company-specific
approach that is driven by
corporate objectives and
strategies with a well-defined
new-product strategy at its core.
100.0%

e. do all of the above.


Score: 2 / 2

Question 15 (2.00 points)
From April 14 through May 23, 2010, KFC set aside 50 cents from each bucket of Original Recipe or
Kentucky Grilled chicken that it sold in the U.S. The funds were designated for a contribution by KFC to
Susan G. Komen for the Cure . This campaign-which was named "Buckets for the Cure-raised a little
over $4.2 million, which was the largest-ever single donation to Susan G. Komen for the Cure. KFCs
efforts to build awareness of the campaign, as well as to increase awareness of breast cancer research-
related issues, included:


> informative television advertising on a national scale


> illumination of the KFC global headquarters building in Louisville with pink floodlights (the official color of
the Komen organization) each night from April 14 through May 23, 2010


> packaging of bucket orders at KFC restaurants in pink commemorative buckets during the course of the
campaign


> linking of the KFC website to an interactive website (http://www.bucketsforthecure.com) where visitors
had the opportunity to learn more about breast cancer research and make online donations


> "Buckets for the Cure-related communication through Facebook ,Twitter, and YouTube
In addition, KFCs "Colonel Sanders mascot was decked out in pink instead of his usual white Palm Beach
suit for all public appearances during the campaign.


"Buckets for the Cure illustrates which of the following concepts?
Student response:
!ercent
Value
Correct
Response
Student
Response
Answer Choices
0.0%

a. Cause-related marketing
True . . . but there's more.
See pp. 252-256, pp. 262-
263, pp. 278-281.


b. Public relations


c. Integrated marketing
communications (IMC)



d. All of the above


e. None of the above


Score: 0 / 2

Question 16 (2.00 points)
With regard to marketing by nonprofit organizations, with which of the following statements would the
authors of your text likely DISAGREE?
Student response:
!ercent
Value
Correct
Response
Student
Response
Answer Choices


a. The target market of a nonprofit
organization may consist of
people who do not care about-
or are even opposed to-
receiving the services that the
organization offers.
0.0%

b. Nonprofit organizations often
market products or services that
are by definition "very low
involvement.
The authors state this. See
pp. 196-199.



c. The marketing mix for nonprofit
organizations consists of 3 Ps:
product, place (distribution), and
promotion.


d. Many nonprofit organizations are
prohibited from advertising. For
these organizations, public
service announcements (PSAs)
may be an important element of
the promotion strategy,.


e. The benefits of nonprofit
organizations product offerings
may be more difficult to
communicate than those of for-
profit marketers.


Score: 0 / 2

Question 17 (2.00 points)
Which of the following is NOT a logistical component of the supply chain?
Student response:
!ercent
Value
Correct
Response
Student
Response
Answer Choices
100.0%

a. Sales forecasting


b. Production scheduling


c. Inventory control


d. Warehousing and materials
handling


e. Order processing


Score: 2 / 2

Question 18 (2.00 points)
_____ is the function that links the consumer, customer, and public to the marketer through information.
Student response:
!ercent
Value
Correct
Response
Student
Response
Answer Choices


a. Artificial intelligence


b. Decision framing


c. A feasibility study
100.0%

d. Marketing research


e. Decision support


Score: 2 / 2

Question 19 (2.00 points)
Unified Grocers, Inc., a Chicago-based grocery wholesale cooperative, provides all kinds of meats, cereals,
canned and fresh fruits and vegetables, beauty aids, health care items, and pet products to the
supermarkets in its channel. Unified is aiding consumers by addressing a ________________.
Student response:
!ercent
Value
Correct
Response
Student
Response
Answer Choices
0.0%

a. spatial discrepancy
No . . . see pp. 201-204.


b. discrepancy of quantity



c. discrepancy of assortment


d. demand discrepancy


e. discrepancy of product
specifications


Score: 0 / 2

Question 20 (2.00 points)
Prof A is listening to the syndicated radio program "Gospel Traxx, hosted by Walt "Baby Love. The show
goes to commercial break, and Prof A hears the following commercial.


Male Voice 1: Yo, you ready to hit Mickey D's?

Male Voice 2: Yeah, but hold up. I'm checking out 365black.com. Can you believe how much business they
do in communities like ours?

Male Voice 1: School me!

Male Voice 2: You know that chocolate sauce on McDonald's sundaes? Well, a black company is one of the
suppliers.

Male Voice 1: For real?

Male Voice 2: For real! They support all kinds of local entrepreneurs, over five billion dollars worth of
support.

Male Voice 1: Billion with a !

Male Voice 2: Yeah! McDonald's really puts their money where our
mouths are.

Male Voice 1: True that!

Announcer: Supporting entrepreneurs and the communities we serve . . . McDonald's!

This McDonalds radio commercial is an example of:
Student response:
!ercent
Value
Correct
Response
Student
Response
Answer Choices
100.0%

a. institutional advertising.


b. comparative advertising.


c. pioneering advertising.


d. scientific advertising execution.


e. a convenience advertising
appeal.


Score: 2 / 2

Question 21 (2.00 points)
When Krispy Kreme decided to expand its operation internationally, it chose Canada as its first new market.
In accordance with the policy of minimizing investment costs and risk, the company sold the right to
manufacture and sell its doughnuts to Canadians. In other words, Krispy Kreme used:
Student response:
!ercent
Value
Correct
Response
Student
Response
Answer Choices


a. contract manufacturing.


b. direct investment.


c. exporting.


d. a joint venture.
100.0%

e. licensing.


Score: 2 / 2

Question 22 (2.00 points)
ntensity or distribution refers to
Student response:
!ercent
Value
Correct
Response
Student
Response
Answer Choices


a. the size of a wholesalers truck
fleet.


b. the number of manufacturers
sales reps employed in a
particular industry.


c. the number of media through
which a brand is advertised.


d. the dollar amount of a
producers and a retailers joint
expenditures of cooperative
advertising for one of the
producers products that the
retailer sells.
100.0%

e. the number of intermediaries
through which a producer
distributes a product within a
particular geographic area.


Score: 2 / 2

Question 23 (2.00 points)
Using the BCG (Boston Consulting Group) matrix, how would Berkshire Hathaway categorize itself?
Student response:
!ercent
Value
Correct
Response
Student
Response
Answer Choices


a. As a dog


b. As a star
0.0%

c. As a cash cow
A diversified "holding"
corporation such as
Berkshire Hathaway WOULD
NOT categorize its overall
organization using the BCG
matrix; it would categorize
each of its individual
subsidiaries/SBUs (e.g.,
GEICO, Justin Brands,
Benjamin Moore & Company,
Fruit of the Loom, Clayton
Homes, Burlington Northern
Santa Fe Corporation, The
!ampered Chef, H.H. Brown
Shoe Company, Helzberg
Diamonds . . . ). See pp. 23-
24.


d. As a problem child



e. As none of the above.


Score: 0 / 2

Question 24 (2.00 points)
Hope Swope works as a salesperson for a company that provides "activity- and nutrition-based employee
wellness programs to medium-sized businesses. When contacting a potential client, she often talks to
several people before she locates someone who can make an authoratative "yes or "no pronouncement
regarding the companys purchase of such a program. Hope is having trouble identifying the ___________.
Student response:
!ercent
Value
Correct
Response
Student
Response
Answer Choices


a. influencer


b. gatekeeper


c. user



d. decider.
0.0%

e. purchaser.
No . . . see pp. 110-111.


Score: 0 / 2

Question 25 (2.00 points)
Even though they do not have authorization from manufacturers, distributors will sometimes divert products
from low-price markets to sell them in high-price markets. For example, a consumer in the U.S. who is
shopping for a new Japanese-built Yamaha grand piano may find one for $27,500 and another very similar
one for $14,000. The difference is that the $27,500 piano was built for the U.S. market; the $14,000 piano
was built for the southeast Asia market and was intended to be sold at a list price of $8,500 in that market.
Through unauthorized diversion to the U.S., some distributor (which has not been authorized by the Yamaha
Corporation) makes additional money, even though the consumer pays less. (The consumer may actually be
ripped off, though, because the wooden parts of the piano have been carefully selected and treated for
optimal performance in the humidity of southeast Asia; those parts may crack or wear out prematurely from
becoming excessively dry in some areas of the U.S.) This scenario is an example of
Student response:
!ercent
Value
Correct
Response
Student
Response
Answer Choices


a. dumping.


b. price fixing.


c. countertrade.
100.0%

d. the gray market.


e. price escalation.


Score: 2 / 2

Question 26 (2.00 points)
Which of the following is CORRECT regarding the product life cycle?
Student response:
!ercent
Value
Correct
Response
Student
Response
Answer Choices


a. Increasing competition,
aggressive brand advertising,
and healthy profits are all
characteristics found in the
introductory stage of the product
life cycle.


b. Profits typically reach their peak
during the maturity stage of the
product life cycle.



c. Almost all purchases in the
maturity and decline stages of
the product life cycle represent
repeat purchasing.


d. The length of the introductory
stage of the product life cycle for
a new product is largely
determined by the political and
legal environment.
0.0%

e. Normally, the longest stage of
the product life cycle is the
growth stage.
No . . . it's the maturity
stage. See pp. 181-184.


Score: 0 / 2

Question 27 (2.00 points)
Nestl offers Stouffer's gourmet entrees for one segment of the frozen dinner market, and Lean Cuisine for
another segment that wants both good taste and low calories. When Nestl chose to serve two well-defined
market segments and develop distinct marketing mixes for each, it was implementing a _____ strategy.
Student response:
!ercent
Value
Correct
Response
Student
Response
Answer Choices


a. market integration


b. concentrated targeting


c. undifferentiated targeting
100.0%

d. multisegment targeting


e. one-to-one marketing


Score: 2 / 2

Question 28 (2.00 points)
On April 7, 2010, Dr. Robert Gerlach appeared on the radio news/talk program DayBreak USA. Dr. Gerlach
is a dentist who is employed by Procter & Gamble. The host of the show, Scott West, interviewed Dr.
Gerlach with regard to the subject of sensitive teeth. In the course of the interview, Dr. Gerlach was given
the opportunity to explain the causes of sensitive teeth. Dr. Gerlach was also able to inform the audience as
to the unique properties and capabilities of the Crest Pro-Health Sensitive Shield brand of toothpaste (a P&G
product). Through its _____________ efforts, Procter & Gamble was successful in securing DayBreak USAs
invitation for Dr. Gerlach to appear on the show. For Procter & Gamble, the broadcast of the radio segment
itself is an example of ________________.
Student response:
!ercent
Value
Correct
Response
Student
Response
Answer Choices



a. public relations; product
publicity


b. guerilla marketing; use of social
media


c. lobbying; sponsorship
0.0%

d. product placement; radio
advertising
No . . . see pp. 250-252, pp.
278-281.


e. viral marketing; consumer
education


Score: 0 / 2

Question 29 (2.00 points)
Peachtree Windows has no windows in inventory waiting for someone to order them. It does not make a
window until it is ordered. It is able to make 27,000 different window configurations and will make special
adjustments to these configurations if there is a request for modifications as long as the modifications meet
the company's exacting safety requirements. Peachtree Window engages in:
Student response:
!ercent
Value
Correct
Response
Student
Response
Answer Choices


a. push distribution


b. niche marketing


c. psychographic segmentation
100.0%

d. mass customization


e. a production orientation


Score: 2 / 2

Question 30 (2.00 points)
Which of the following mission statements might the authors of your text say is too broad?
Student response:
!ercent
Value
Correct
Response
Student
Response
Answer Choices
100.0%

a. "To equip people to make a
difference in their world-we do
this by creating outstanding
products and by trying to make
a difference in the communities
in which we live and work
(Timberland).


b. "To be a trusted, respected, and
ethical food industry leader that
excels at bringing delicious and
nutritious meat and specialty
products to millions every day
while setting industry standards
for corporate social
responsibility (Smithfield
Foods)


c. "To facilitate the security of our
members, associates, and their
families through provision of a
full range of highly competitive
financial products and services;
in so doing, we seek to be the
provider of choice for the
military community. (USAA)


d. "To provide personal vehicle
owners and enthusiasts with the
vehicle related products and
knowledge that fulfill their wants
and needs at the right price. Our
friendly, knowledgeable and
professional staff will help
educate, inspire and problem-
solve for our customers.
(Advance Auto Parts)


e. "To offer lending and investment
products that serve low- and
moderate-income individuals
and families; to improve
underserved low- and moderate-
income communities; to create
sustainable practices in the long
haul (Bank of America)


Score: 2 / 2

Question 31 (2.00 points)
Regarding the transportation component of supply chain logistics, when we refer to "reliability, we are
talking about:
Student response:
!ercent
Value
Correct
Response
Student
Response
Answer Choices


a. a carrier's ability to provide the
appropriate equipment and
conditions for moving specific
kinds of goods.
100.0%

b. the consistency with which a
carrier delivers goods on time.


c. a carrier's ability to move goods
over a specific route.


d. the relative ease with which a
shipment can be located and
transferred.


e. the total amount a specific
carrier charges to move the
product.


Score: 2 / 2

Question 32 (2.00 points)
Marriott International uses blogging to communicate with customers online. Marriotts chairman and CEO Bill
Marriott-son of founder J.W. Marriott-began a blog about the company in January, 2007. Mr. Marriott
generally posts a down-to-earth, sentimental message once a week or so. Once readers realized it really was
Bill Marriott and not an advertising ploy, they quickly warmed to the idea and appreciated his candid
comments about Marriott properties and the companys activities. This is an example of:
Student response:
!ercent
Value
Correct
Response
Student
Response
Answer Choices



a. a corporate blog.
0.0%

b. a nonmarketing-controlled
information source
No . . . see pp. 252-256.


c. syndicated research.


d. competitive intelligence
gathering


e. a noncorporate blog.


Score: 0 / 2

Question 33 (2.00 points)
The buyer who heads a department in a department store is responsible for the:
Student response:
!ercent
Value
Correct
Response
Student
Response
Answer Choices


a. stores exchange and return
policies.


b. stores credit policies.


c. chains growth in number of
locations.
100.0%

d. merchandise mix of that
department.


e. chainwide advertising
campaigns.


Score: 2 / 2

Question 34 (2.00 points)
If the adminstration of West Virginia State University asked you to write a marketing plan for the
university-and you were working on the SWOT analysis part of the plan-how would you categorize
increasing textbook prices, tightening of credit by banks and other lending institutions, and a potential
government-mandated increase in tuition rates?
Student response:
!ercent
Value
Correct
Response
Student
Response
Answer Choices


a. As an opportunity


b. As weaknesses


c. As strengths
100.0%

d. As a threat


e. As collateral damage


Score: 2 / 2

Question 35 (2.00 points)
The 2011 Ford Harley-Davidson F-150 truck is an example of
Student response:
!ercent
Value
Correct
Response
Student
Response
Answer Choices


a. private-label branding.


b. complementary branding.


c. fusion of innovation.


d. piggyback branding.
100.0%

e. cobranding.


Score: 2 / 2

Question 36 (2.00 points)
The stages of the consumer decision making process are, in their proper order:
Student response:
!ercent
Value
Correct
Response
Student
Response
Answer Choices


a. problem recognition >>
determine product specifications
>> search for suppliers >> seek
sales proposals/responses to
RFP >> purchase decision >>
post-purchase evaluation


b. information search >> need
recognition >> evaluation of
alternatives >> purchase >>
post-purchase behavior


c. evaluation of alternatives >>
need recognition >> information
search >> post-search behavior
>> purchase
100.0%

d. need recognition >> information
search >> evaluation of
alternatives >> purchase >>
post-purchase behavior


e. need recognition >> dissonance
>> evaluation of alternatives
>> information search >>
purchase


Score: 2 / 2

Question 37 (2.00 points)
To overcome low redemption rates for coupons, marketers are:
Student response:
!ercent
Value
Correct
Response
Student
Response
Answer Choices


a. issuing fewer coupons and using
more EDLP (everyday low
pricing).


b. shortening the time the coupon
can be redeemed, in order to
create a feeling of urgency
among consumers.


c. using instant coupons on
product packages.


d. distributing single all-purpose
coupons that can be redeemed
for several brands.
100.0%

e. doing all of the above.


Score: 2 / 2

Question 38 (2.00 points)
Retailers control the six Ps of the retailing mix. The six Ps include the four Ps of the marketing mix PLUS:
Student response:
!ercent
Value
Correct
Response
Student
Response
Answer Choices


a. policies and procedures.


b. power and practicality.


c. proprietor and principal.
100.0%

d. personnel and presentation.


e. perishability and permanence.


Score: 2 / 2

Question 39 (2.00 points)
A(n) ___________________ is defined as a set of unique features of a company and its products that are
perceived by the target market as significant and superior to the competition.
Student response:
!ercent
Value
Correct
Response
Student
Response
Answer Choices


a. evoked set


b. universal product code (UPC)
100.0%

c. competitive advantage


d. temporal discrepancy


e. touch point


Score: 2 / 2

Question 40 (2.00 points)
A reading service for the visually impaired requires each reader applicant to prepare and submit a one-hour
audition tape. Material for the taping is chosen by the service. The goal of the service is to hire readers
whose voices are clearly audible, pleasant, and accent free so that it is not necessary to assign the same
reader to a client on each reading occasion. The reading service is trying to minimize problems that are
associated with which characteristic of services?
Student response:
!ercent
Value
Correct
Response
Student
Response
Answer Choices


a. Intangibility


b. Inseparability
100.0%

c. Heterogeneity


d. Perishability


e. Intractability


Score: 2 / 2

Question 41 (2.00 points)
A management decision problem ____________________; by contrast, a marketing research problem
____________________.
Student response:
!ercent
Value
Correct
Response
Student
Response
Answer Choices


a. is narrow and specific; is broad
and general


b. involves determining what
information is needed; is often
solved by identifying a series of
management decision problems
100.0%

c. is action oriented; is information
oriented


d. is dichotomous; is autonomous


e. is synergistic; is pervasive


Score: 2 / 2

Question 42 (2.00 points)
Retailers and merchant wholesalers are examples of intermediaries that:
Student response:
!ercent
Value
Correct
Response
Student
Response
Answer Choices
100.0%

a. take title to the products they
handle.


b. create spatial and temporal
discrepancies.


c. exist only in socialist (collective)
economies, and are not found in
capitalistic (market) economies.


d. reduce the contact efficiency of
distribution channels.


e. eliminate the need for physical
transportation of products.


Score: 2 / 2

Question 43 (2.00 points)
A marketing executive who believes in the "total CSR approach to social responsibility will adopt the
perspective of the "pyramid of corporate social responsibility, which contains all of the following
components EXCEPT:
Student response:
!ercent
Value
Correct
Response
Student
Response
Answer Choices


a. economic responsibility.


b. legal responsibility.
100.0%

c. cultural responsibility.


d. ethical responsibility.


e. philanthropic responsibility.


Score: 2 / 2

Question 44 (2.00 points)
Which of the following is NOT true with regard to consumer involvement in the purchase process?
Student response:
!ercent
Value
Correct
Response
Student
Response
Answer Choices


a. If a purchase is a high-
involvement purchase, the
consumer will probably engage
in extensive information
search-both internal and
external-before deciding which
brand to buy.
100.0%

b. Marketers can never raise
consumers involvement level in
the purchase of low-involvement
products.


c. The circumstances of a purchase
may temporarily transform a
low-involvement decision into a
high-involvement decision.


d. Some product and service
categories-such as homes,
automobiles, fashion, and choice
of a college or university to
attend-tend to be high-
involvement decisions for most
consumers, but there usually
are a few purchasers who treat
the decision as a low-
involvement decision.


e. If a consumer experiences
cognitive dissonance after
purchasing a product, the
purchase decision was probably
a high-involvement one.


Score: 2 / 2

Question 45 (2.00 points)
Which of the following statements about comparative advertising is TRUE?
Student response:
!ercent
Value
Correct
Response
Student
Response
Answer Choices


a. Comparative advertising is
highly effective in Arabic
countries.


b. The FCC (Federal
Communications Commission) is
the only federal agency that has
any regulatory power over
comparative advertising.


c. Brands that are experiencing
strong growth and that tend to
be "the industry standard are
the most likely to use
comparative advertising.



d. Comparative advertising is
regulated in the U.S. and is
completely illegal in some other
countries.
0.0%

e. None of the above.
There IS a valid statement
somewhere among ""a
through "d". !romise. See
pp. 267-268.


Score: 0 / 2

Question 46 (2.00 points)
Razor scooters gained instant popularity because walkers and runners frequently saw people on scooters
zipping by them and having fun. This information suggests that adoption of the Razor scooter was most
affected by which product characteristic?
Student response:
!ercent
Value
Correct
Response
Student
Response
Answer Choices


a. Complexity


b. Trialability


c. Relative advantage
100.0%

d. Obervability


e. Compatibility


Score: 2 / 2

Question 47 (2.00 points)
A "sales-oriented company or organization:
Student response:
!ercent
Value
Correct
Response
Student
Response
Answer Choices


a. defines its business (or mission)
in terms of the benefits its
customers seek.
100.0%

b. seeks to generate sales volume
through intensive promotional
activities-mainly personal
selling and advertising.


c. recognizes that different
customer groups want different
features or benefits.


d. seeks to make a profit by
creating customer value,
providing customer satisfaction,
and building long-term
relationships with customers.


e. focuses on making what the
customer wants, not selling
what the organization wants to
make.


Score: 2 / 2

Question 48 (2.00 points)
Global marketing standardization:
Student response:
!ercent
Value
Correct
Response
Student
Response
Answer Choices


a. is becoming less popular with
large multinationals.


b. emphasizes variations in
product, packaging, and
advertising for each nation or
local market.


c. increases production costs.
100.0%

d. presumes that markets
throughout the world are
becoming more alike.


e. is more widespread in the
marketing of consumer products
than in the marketing of
industrial products.


Score: 2 / 2

Question 49 (2.00 points)
The authors of your text use the "gap model to illustrate discrepancies that can cause problems in service
delivery. These same discrepancies can influence customer evaluations of service quality in a negative way.
Which of the following is NOT one of the gaps in the gap model?
Student response:
!ercent
Value
Correct
Response
Student
Response
Answer Choices


a. The gap between what the
company provides and what the
customer is told it provides


b. The gap between what
management thinks customers
want and the quality
specifications management
develops


c. The gap between the service
quality specifications and the
service that is actually provided


d. The gap between the quality of
services customers want and the
quality of service they receive
100.0%

e. The gap between the level of
service that is provided by a
union employee and the level of
service that is provided by a
nonunion employee


Score: 2 / 2

Question 50 (2.00 points)
When a manufacturer (such as Colgate-Palmolive) splits the cost of advertising with a local retailer (such as
Target) for one of its products (such as Lady Speed Stick), it is called ______________. This form of
advertising is most likely to employ these media: _____________ and/or _____________.
Student response:
!ercent
Value
Correct
Response
Student
Response
Answer Choices


a. image; television; outdoor
100.0%

b. cooperative; newspaper; radio


c. reminder; Internet; alternative


d. primary; magazines; television


e. institutional; outdoor;
magazines


Score: 2 / 2

Question 51 (2.00 points)
According to the criterion of ________________, a selected market segment must be large enough to
warrant developing and maintaining a special marketing mix. Serving the specific needs of this segment
must be commercially viable, even if the number of potential customers is small.
Student response:
!ercent
Value
Correct
Response
Student
Response
Answer Choices


a. causality


b. accountability


c. responsiveness


d. accessibility
100.0%

e. substantiality


Score: 2 / 2

Question 52 (2.00 points)
The appearance of Under Armour in the movie "Blind Side, the Cadillac CTS and Escalade ETX in "The
Matrix Reloaded, and Heineken in "Casino Royale are examples of:
Student response:
!ercent
Value
Correct
Response
Student
Response
Answer Choices


a. cobranding.


b. commercialization.


c. diffusion of innovation.
100.0%

d. product placement.


e. advocacy advertising.


Score: 2 / 2

Question 53 (2.00 points)
Coca-Cola, Planters, and Purina One are _____________ brands; Big K (sold by Kroger), Archer Farms
(sold by Target), and Ol Roy (sold by Walmart) are ____________ brands, also sometimes called
___________ brands.
Student response:
!ercent
Value
Correct
Response
Student
Response
Answer Choices


a. generic; knockoff;
manufacturers


b. private-label; gray-market;
generic


c. house; manufacturers; private-
label
100.0%

d. manufacturers; private-label;
store


e. step-down; generic; gray-
market


Score: 2 / 2

Question 54 (2.00 points)
Services sometimes have important characteristics that customers cannot evaluate-even after purchase-
because they do not have the knowledge or experience to judge those characteristics accurately. The
authors of your textbook refer to such characteristics as
Student response:
!ercent
Value
Correct
Response
Student
Response
Answer Choices


a. search qualities


b. tandem qualities


c. experience qualities
100.0%

d. credence qualities


e. salient qualities


Score: 2 / 2

Question 55 (2.00 points)
All other things being equal, when brand awareness is high, and the brand is perceived to be of high quality,
and customers of the brand are extremely loyal to it, the brand can be said to have __________.
Student response:
!ercent
Value
Correct
Response
Student
Response
Answer Choices
100.0%

a. high brand equity


b. a monopoly


c. brand security


d. synonymity


e. economies of scale


Score: 2 / 2

Question 56 (2.00 points)
The two most expensive kinds of survey research are in-at-home personal interviews (where researchers
send interviewers to visit consumers in their homes and ask them a planned set of questions) and
______________.
Student response:
!ercent
Value
Correct
Response
Student
Response
Answer Choices


a. mail surveys


b. focus groups


c. CLT (central-location telephone)
interviews


d. mall intercept interviews
100.0%

e. executive interviews


Score: 2 / 2

Question 57 (2.00 points)
Sherwin-Williams Paint Company sells the paint it manufactures through its company-owned retail paint
stores. The authors of your text refer to this type of channel of distribution as a(n) ________ ________.
Student response:
!ercent
Value
Correct
Response
Student
Response
Answer Choices
100.0%

a. direct channel


b. retailer channel


c. wholesaler channel


d. agent/broker channel


e. intermediated channel


Score: 2 / 2

Question 58 (2.00 points)
Which of the following does NOT describe the business market, as compared to the consumer market?
Student response:
!ercent
Value
Correct
Response
Student
Response
Answer Choices
100.0%

a. A business buyer is much less
likely to try to negotiate a price
than is a consumer.


b. Business customers tend to be
more geographically
concentrated, and customers in
consumer markets tend to be
more geographically dispersed.


c. Business markets are served by
a more direct channel of
distribution than are consumer
markets.


d. Purchasing by businesses is a
more formal process than
purchasing by consumers.


e. Business marketers tend to have
far fewer customers than
consumer marketers.


Score: 2 / 2

Question 59 (2.00 points)
In the past year, Prof. A has redeemed Speedway "Speedy Rewards points for a $50 gas card and two $25
gas cards, "HiltonHHonors points for a four free stays at Hampton Inn, Southwest Airlines "Rapid Rewards
points for five round-trip tickets from Louisville to Los Angeles, "Choice Privileges points for nine free stays
at Sleep Inn, "Priority Club points for six free stays at Holiday Inn Express, and "MyStarbucks Rewards
points for eighteen free coffee drinks at various Starbucks locations. These transactions indicate that Prof. A
participates in
Student response:
!ercent
Value
Correct
Response
Student
Response
Answer Choices


a. guerilla marketing.


b. viral marketing.
100.0%

c. loyalty programs.


d. experiential retailing.


e. None of the above.


Score: 2 / 2

Question 60 (2.00 points)
Rush Express, a North American delivery service, wants to expand service into South America. If it were to
do so, such an expansion would be classified as ___________ in Ansoffs Product-Market Matrix.
Student response:
!ercent
Value
Correct
Response
Student
Response
Answer Choices


a. diversification


b. market penetration


c. product development
100.0%

d. market development


e. market annexation


Score: 2 / 2

Question 1 (0.10 points)
With regard to social media, with which of the following statements would the authors of your text likely
AGREE?
Student response:
!ercent
Value
Student
Response
Answer Choices


a. Word-of-mouth communication among
consumers has become even more valuable
to marketers as a result of recent advances
in social media technology.


b. Lady Gaga has leveraged social media more
effectively than most-if not all-marketers
of consumer packaged goods.


c. Social media allows marketers to forge
deeper one-on-one relationships with
consumers; it also allows consumers to
connect with each other and share opinions.
100.0%

d. All of the above.


e. None of the above.


Score: 0.1 / 0.1

Question 2 (0.10 points)
Whitney Pitney is shopping for a pair of Nikes at Zappos.com. While she is narrowing her search, she sees a
featured pair of Via Spiga heels. Whitney believes that her best friend forever-Valerie Mallory-definitely
needs to be made aware of the Via Spigas. Whitney clicks a little e-mail icon next to the word "share, and
she then enters Valeries e-mail address and a short personal message suggesting that Valerie may want the
shoes to wear with the dress that she just bought to wear to an upcoming New Years Eve party. The
authors of your text would refer to Whitneys efforts to share information with Valerie (from the perspective
of Zappos) as:
Student response:
!ercent
Value
Student
Response
Answer Choices


a. internal information search.


b. paid media.


c. trade sales promotion.


d. owned media.
100.0%

e. earned media.


Score: 0.1 / 0.1

Question 3 (0.10 points)
With regard to creating and leveraging a social media campaign, with which of the following statements
would the authors of your text likely AGREE?
Student response:
!ercent
Value
Student
Response
Answer Choices


a. The methods that marketers have used in
the past to gain publicity through traditional
media-such as radio, TV, and newspapers-
can also be used to gain publicity through
social media. In other words, the media may
be new, but the methods and techniques are
not.


b. The first step in initiating a social media
campaign is the same as for all other
marketing processes: develop a list of
objectives.


c. Remember that reaching the mass audience
with a static message-such as a national
television advertising campaign-is still the
most cost-effective means of communicating
with the target market and the most
persuasive.


d. All of the above.
100.0%

e. None of the above.


Score: 0.1 / 0.1

Question 4 (0.10 points)
Sondra Alondra owns Gothic Ghoullery, a specialty apparel and accessories retailer with locations in three
regional malls. As of Monday, July 18, Gothic Ghoullerys Facebook page has amassed 12,271 likes. From
this information, Sondra should conclude
Student response:
!ercent
Value
Student
Response
Answer Choices


a. that her companys social media presence is
lacking.


b. that she needs to invest in traditional
media-particularly radio-to drive more
traffic to her companys Facebook page.


c. that she needs to set up individual Facebook
pages for each location, instead of relying
upon one page for the chain as a whole.


d. all of the above.
100.0%

e. none of the above.


Score: 0.1 / 0.1

Question 5 (0.10 points)
Gowalla, Loopt, and Foursquare are categorized by the authors of your text as
Student response:
!ercent
Value
Student
Response
Answer Choices


a. social news sites.


b. virtual worlds.
100.0%

c. location-based social networking sites.


d. microblogs.


e. social media measurement and monitoring
tools.


Score: 0.1 / 0.1
Question 1 (0.10 points)
Best Buy recently instituted a(n) _____________ focus in all of its stores. Each store will customize their
product offerings for the five key customer segments that Best Buy has identified: affluent professional
males, young entertainment enthusiasts, upscale suburban moms, families who are practical technology
adopters, and small businesses with fewer than 20 employees.
Student response:
!ercent
Value
Student
Response
Answer Choices


a. ethnocentric


b. production-oriented


c. channel-based


d. trade-oriented
100.0%

e. customer-centric


Score: 0.1 / 0.1

Question 2 (0.10 points)
The guest at a Marriott hotel stepped in a mud puddle on his way into the hotel. His shoes were drenched,
and he had planned to wear them to an important sales presentation in an hour. The hotel concierge saw the
guest's unhappy face and asked if he could help. After learning of the problem, the concierge loaned the
guest his own shoes to wear to the meeting. The concierge was able to handle the situation in a matter that
created customer satisfaction because Marriott:
Student response:
!ercent
Value
Student
Response
Answer Choices


a. has a sales orientation.


b. uses knowledge management.


c. limits customer interactions.
100.0%

d. empowers its employees.


e. takes a highly-centralized, standardized
approach to guest satisfaction.


Score: 0.1 / 0.1

Question 3 (0.10 points)
The Nokia Flagship Store in Midtown Manhattan boasts 2,000-square-feet of floor space with minimalist
displays stretched along the walls. Here consumers can use interactive visuals to change or add text to
messages via the products nestled below. "Experience Areas" feature phones connected to photo printers,
speakers, notebook computers, and Bluetooth headsets to demonstrate the interactivity and full range of
features available on the cell phones. These "Experience Areas" are examples of _____ where customers can
interact with the technology and provide information to Nokia.
Student response:
!ercent
Value
Student
Response
Answer Choices


a. data mines


b. focus zones


c. experiment stations


d. propaganda stations
100.0%

e. touch points


Score: 0.1 / 0.1

Question 4 (0.10 points)
When Janis Moranis called to place a catalog order for a humidor for her father, she was pleased when the
operator suggested that she might also be interested in a subscription to a magazine targeted to cigar
lovers. The operator was using _____--a method commonly used to leverage customer information.
Student response:
!ercent
Value
Student
Response
Answer Choices


a. data mining
100.0%

b. cross-selling


c. trading up


d. psychographic aggregation


e. keystoning


Score: 0.1 / 0.1

Question 5 (0.10 points)
After Gary and Shari Perry had paid in advance for their upcoming one-week stay at a small Vermont inn,
Shari began to feel that they might have chosen poorly and that they were committing themselves to stay at
a place they might not enjoy. Then Shari got a letter from the hotel's owner stating that she (i.e., the
owner) was looking forward to their visit and asking what they most enjoyed for breakfast. The hospitality of
the letter dealt with Shari's cognitive dissonance by:
Student response:
!ercent
Value
Student
Response
Answer Choices


a. cross-selling.


b. disintermediation.


c. spamming and flaming.


d. playing hardball.
100.0%

e. reinforcing her purchase decision.


Score: 0.1 / 0.1


Total score: 0.5 / 0.5 = 100.0%
When the Mosquito Magnet was introduced, it was designed to rid the immediate area of mosquitoes and
other annoying insects. The technology for the Mosquito Magnet had taken years to develop. It is a patented
grill-like apparatus that emits carbon dioxide to attract bugs to a fan that draws them into the device where
they die. What type of pricing policy would you recommend the company use to introduce this product to the
market?
Student response:
!ercent Student Answer Choices
Value Response


a. Status quo pricing


b. Penetration pricing
100.0%

c. Price skimming


d. Flexible pricing


e. Leader pricing


Score: 0.1 / 0.1

Question 2 (0.10 points)
Jones Soda Company and Big Sky Brands have introduced Jones Soda Carbonated Candy, a candy that
delivers a blast of the most popular Jones Soda flavors along with an oddly enjoyable tongue-tingling
sensation. A _____ strategy would most likely be used with this product to convince consumers to try it and
not buy some other brand.
Student response:
!ercent
Value
Student
Response
Answer Choices


a. price-skimming
100.0%

b. penetration pricing


c. status quo pricing


d. cost bundling


e. price-lining


Score: 0.1 / 0.1

Question 3 (0.10 points)
___________ occurs when a firm is customer driven and seeks to understand the attributes customers want
in the goods and services they buy and the value of those attributes to customers. Thus, the price of the
product is set at a level that seems to the customer to be a good price compared with the prices of other
options.
Student response:
!ercent
Value
Student
Response
Answer Choices


a. Noncumulative pricing


b. Market-concept pricing


c. Price bundling


d. Bait pricing
100.0%

e. Value-based pricing


Score: 0.1 / 0.1

Question 4 (0.10 points)
A price tactic that requires the purchaser to absorb the freight costs from the shipping point is called _____.
In this case, the farther buyers are from sellers, the more they pay because transportation costs generally
increase with the distance merchandise is shipped.
Student response:
!ercent
Value
Student
Response
Answer Choices


a. base-point pricing


b. zone pricing


c. uniform delivered pricing


d. freight absorption pricing
100.0%

e. FOB origin pricing


Score: 0.1 / 0.1

Question 5 (0.10 points)
If a company decides to divide its market area into segments or regions and charge a flat rate for freight to
all customers in a given region, the company is using _____ pricing.
Student response:
!ercent
Value
Student
Response
Answer Choices
100.0%

a. zone


b. uniform delivered


c. freight absorption


d. FOB origin


e. basing-point


Score: 0.1 / 0.1
Question 1 (0.10 points)
An organization is using _____ when it sets its prices so that total revenue is as large as possible relative to
total costs.
Student response:
!ercent
Value
Student
Response
Answer Choices
100.0%

a. profit maximization


b. market share pricing


c. demand-oriented pricing


d. sales maximization


e. status quo pricing


Score: 0.1 / 0.1

Question 2 (0.10 points)
________________ measures the overall effectiveness of management in generating profits with its
available assets.
Student response:
!ercent
Value
Student
Response
Answer Choices
100.0%

a. Return on investment (ROI)


b. Economic order quantity (EOQ)


c. Retained earnings


d. Index of retail saturation (IRS)


e. Buying Power Index (BPI)


Score: 0.1 / 0.1

Question 3 (0.10 points)
Queeg Industries sells all types of artists' paintbrushes. When the owner of Queeg learned that Patterson
Art-one of its biggest competitors in the Northeast region-had lowered its prices on all synthetic brushes
by 5 percent, he did the same. This is an example of _____ pricing.
Student response:
!ercent
Value
Student
Response
Answer Choices
100.0%

a. status quo


b. target return


c. market share


d. predatory


e. cost-plus


Score: 0.1 / 0.1

Question 4 (0.10 points)
During basketball games at his hometowns high school, Scooter Tudor runs a concession stand. The costs
associated with the purchase of hot dogs, mustard, relish, ketchup, chips, sodas, paper napkins, and cups
are all examples of _____ costs.
Student response:
!ercent
Value
Student
Response
Answer Choices


a. marginal
100.0%

b. variable


c. fixed


d. promotional


e. liquidation


Score: 0.1 / 0.1

Question 5 (0.10 points)
__________ is the practice of marking up prices to a level that is double the wholesale price.
Student response:
!ercent
Value
Student
Response
Answer Choices


a. Margin pricing
100.0%

b. Keystoning


c. Break-even pricing


d. Symmetrical pricing


e. Central-point pricing


Score: 0.1 / 0.1
Question 1 (0.10 points)
________________ consists of all marketing activities that stimulate consumer purchasing, such as
coupons, contests, free samples, and trade shows.
Student response:
!ercent
Value
Student
Response
Answer Choices


a. Channel cooperation


b. Push strategy


c. Cooperative advertising
100.0%

d. Sales promotion


e. Personal selling


Score: 0.1 / 0.1

Question 2 (0.10 points)
Why do some marketers offer rebates instead of price reductions to induce short-term sales?
Student response:
!ercent
Value
Student
Response
Answer Choices
100.0%

a. Rebates are especially good at enticing
purchases, but most consumers never
bother to redeem them.


b. A price reduction of this type would likely be
an example of price discrimination.


c. Rebates offer a more immediate reward
than price reductions.


d. Rebates have a much higher redemption
rate than coupons.


e. Rebates result in brand-loyal consumers.


Score: 0.1 / 0.1

Question 3 (0.10 points)
Unlike other sales promotion activities, the objective of a loyalty marketing program is to:
Student response:
!ercent
Value
Student
Response
Answer Choices


a. increase the frequency with which
consumers switch brands.


b. appeal to bargain hunters who consistently
buy the lowest-priced brand.


c. modify customers attitudes toward the
brand.
100.0%

d. build long-term, mutually beneficial
relationships between a company and its
key customers.


e. engage in price discrimination without
violating existing laws.


Score: 0.1 / 0.1

Question 4 (0.10 points)
When Porter McWhorter went to Target to buy a 4-pack of Gillette Fusion razor blade cartridges, he found
something he hadnt seen before: a 4-pack of cartridges that included, in the same package, a 1.7-ounce
bottle of Gillette body wash. The price of the 4-pack with the trial-sized bottle was the same as the price
that Porter was accustomed to paying for a 4-pack of cartridges. The package labeling made it clear that the
extra value was a limited-time offering. Gillettes limited-time 4-pack of blades + 1.7-ounce body wash
package is an example of:
Student response:
!ercent
Value
Student
Response
Answer Choices


a. consumer sales promotion.


b. a premium.


c. sampling.
100.0%

d. all of the above.


e. none of the above.


Score: 0.1 / 0.1

Question 5 (0.10 points)
With regard to relationship selling (also called consultative selling), which of the following is TRUE?
Student response:
!ercent
Value
Student
Response
Answer Choices


a. Relationship selling is more common in the
selling of industrial goods than in the selling
of consumer goods.


b. The emphasis is on the salespersons role as
"consultant, partner, and problem-solver
with and for their customers.


c. A salesperson whose moral character is at
the preconventional level most likely is not
truly a "relationship salesperson.


d. Relationship selling is a multistage process
that develops prospects into long-term,
satisfied customers.
100.0%

e. All of the above.


Score: 0.1 / 0.1
Frito-Lay has run television and print commercials in which it shows that consumers prefer Frito's Stax crisps
to Pringles (formerly a brand of Procter & Gamble, now owned by Diamond Foods). This is an example of
_____ advertising.
Student response:
!ercent
Value
Student
Response
Answer Choices


a. advocacy


b. institutional
100.0%

c. comparative


d. image


e. pioneering


Score: 0.1 / 0.1

Question 2 (0.10 points)
The following message appears on a billboard in Lexington, Kentucky: "Pls dnt txt wyl u drv;-). The simple
ad design includes the message in plain, black letters against a yellow background, with the name and logo
of Kentucky Farm Bureau (which is an insurance company) in the lower right-hand corner. This billboard ad
is an example of
Student response:
!ercent
Value
Student
Response
Answer Choices


a. cooperative advertising.


b. comparative advertising.


c. disintermediation.
100.0%

d. advocacy advertising.


e. product placement.


Score: 0.1 / 0.1

Question 3 (0.10 points)
The first step in the advertising campaign decision process is to:
Student response:
!ercent
Value
Student
Response
Answer Choices


a. choose media.


b. evaluate the impact of the campaign.
100.0%

c. determine the objectives of the campaign.


d. write the ad copy.


e. choose an executional style.


Score: 0.1 / 0.1

Question 4 (0.10 points)
______________ executional style is used more often in radio and television commercials than in
newspaper or magazine advertising. It is particularly popular among advertisers of products that tend to be
purchased with limited problem solving or routine-response behavior, such as candy, gum, or cigarettes.
Student response:
!ercent
Value
Student
Response
Answer Choices


a. musical


b. scientific


c. demonstration


d. spokesperson/testimonial
100.0%

e. humorous


Score: 0.1 / 0.1

Question 5 (0.10 points)
Patagonia (a retailer that specializes in outdoor clothing and related gear) sells a line of t-shirts that feature
the artwork of nature conservationist James Prosek. Each of the shirts is imprinted with one of Mr. Proseks
drawings of a trout. For each trout t-shirt sold, Patagonia donates $5 to a grassroots organization that works
to protect trout species and their natural habitat. This action by Patagonia would be categorized by the
authors of your textbook as:
Student response:
!ercent
Value
Student
Response
Answer Choices


a. institutional advertising.
100.0%

b. cause-related marketing.


c. a joint venture.


d. environmental management.


e. cobranding.


Score: 0.1 / 0.1
Question 1 (.1 points)
Every year, the Discovery Channel has what it calls Shark Week, a weeklong marathon of programs on
sharks. In New York, Discovery Channel street teams disguised as Surfers, Bight University faculty, and
Bight University "chewleaders" will attack city streets in July visiting morning TV shows, landmarks, and
high-traffic areas to promote the series. An article in the ew York Times on Shark Week and an interview of
street team members on a morning TV show would be examples of:
Student response:
!ercent
Value
Student
Response
Answer Choices


a. sales promotion.


b. advertising.


c. personal sales presentations.
100.0%

d. publicity.


e. advergaming.


Score: 0.1 / 0.1

Question 2 (.1 points)
Which of the following statements about the characteristics of the elements in the promotional mix is TRUE?
Student response:
!ercent
Value
Student
Response
Answer Choices


a. Public relations gives greater control over
message content than does advertising.


b. Personal selling is the most efficient way to
reach a large audience.
100.0%

c. Compared to advertising and public
relations, personal selling provides
marketers with more rapid feedback.


d. Sales promotion usually involves direct
communication that is tailored to the
individual recipient.


e. All of the above.


Score: 0.1 / 0.1

Question 3 (.1 points)
Marketing decision makers base many of their promotion-related decisions upon a classic model of
persuasion that is closely tied to our understanding of consumer behavior: the "AIDA model. The authors of
your text state that most consumers who are involved in a high-involvement purchase decision will pass
through the four stages of the model. "AIDA stands for:
Student response:
!ercent
Value
Student
Response
Answer Choices


a. anticipation, investigation, decision,
acquisition


b. aggravation, introspection, dissonance,
agreement


c. awareness, identification, differentiation,
approval


d. accumulation, internalization, distrust,
acceptance
100.0%

e. attention, interest, desire, action


Score: 0.1 / 0.1

Question 4 (.1 points)
The promotional mix elements that should be emphasized for products moving into the growth stage of the
product life cycle are:
Student response:
!ercent
Value
Student
Response
Answer Choices


a. public relations and reminder advertising.


b. sales promotion and public relations.
100.0%

c. personal selling and persuasive advertising.


d. sales promotion and informative advertising.


e. sales promotion only.


Score: 0.1 / 0.1

Question 5 (.1 points)
In summer of 2009, Del Monte Foods (parent of, among others, the Gravy Train, Pup-Peroni, Meaty Bone,
Jerky Treats, Kibbles `n Bits, 9 Lives, Pounce, and Milk-Bone brands) launched a multifaceted campaign
featuring its Meow Mix brand of cat food and the advertising tag line "Think like a cat. The campaign
culminated in a 30-minute program, aired on GSN (the Game Show Network), in which eight "cat-testants
and their owners competed for a $25,000 prize (with a $1 million bonus round).


In the weeks leading up to the contest, auditions were promoted through national television, through the
Internet, through local radio, "on pack (i.e., on Meow Mix packages), on FSIs, through local radio and
newspaper, and through pet shelters. As the cattestant-and-human-companion teams were narrowed from a
field of over 10,000 to the eight finalists for the game show, each team received a gift bag filled with Meow
Mix samples. The Hollywood premiere, which took place three days before the show, featured such game
show personalities as Monty Hall (Lets Make a Deal), Peter Marshall (Hollywood Squares), Richard Karn
(Family Feud), and Ken Jennings (all-time Jeopardy champion), with celebrities such as Stacey Kiebler
(Dancing with the Stars), Michelle Trachtenburg (Buffy the Vampire Slayer and Gossip Girl), and Alfonso
Ribiero (Fresh Prince of Bel-Air) also in attendance . . . with catered delicacies such as tuna tartare, provided
by Wolfgang Puck.


Which of the following labels BEST describes this campaign?
Student response:
!ercent
Value
Student
Response
Answer Choices


a. Advertising


b. Public relations


c. Sales promotion


d. Personal selling
100.0%

e. Integrated marketing communications (IMC)


Score: 0.1 / 0.1
Each department in a department store is usually headed by a _____________ who selects s the
merchandise mix for the department and may choose the promotional devices and personnel for that
department.
Student response:
!ercent
Value
Student
Response
Answer Choices


a. franchisee


b. comptroller
100.0%

c. buyer


d. human resource manager


e. retail analyst


Score: 0.1 / 0.1

Question 2 (0.10 points)
Best Buy is a
Student response:
!ercent
Value
Student
Response
Answer Choices


a. department store.


b. convenience store.


c. full-line discount store.


d. supercenter.
100.0%

e. specialty discount store.


Score: 0.1 / 0.1

Question 3 (0.10 points)
Kroger is a
Student response:
!ercent
Value
Student
Response
Answer Choices


a. department store.


b. convenience store.


c. full-line discount store.
100.0%

d. supermarket.


e. specialty discount store.


Score: 0.1 / 0.1

Question 4 (0.10 points)
Walmart is a(n)
Student response:
!ercent
Value
Student
Response
Answer Choices
100.0%

a. full-line discount store.


b. convenience store.


c. department store.


d. category killer.


e. off-price retailer.


Score: 0.1 / 0.1

Question 5 (0.10 points)
Large specialty stores such as Office Depot, Bed Bath & Beyond, Best Buy, and Toys `R Us are often referred
to as _____________ because they can destroy the profit potential of an entire category of merchandise for
other retailers.
Student response:
!ercent
Value
Student
Response
Answer Choices


a. anchor stores
100.0%

b. category killers


c. boutiques


d. hypermarkets


e. scrambled merchandisers


Score: 0.1 / 0.1

Question 6 (0.10 points)
Well-known manufacturers such as Nike, Liz Claiborne, and Mikasa have decided that the most profitable
way to dispose of out-of-season and irregular stock is to open stores and sell their own merchandise. These
stores are a type of:
Student response:
!ercent
Value
Student
Response
Answer Choices


a. seasonal outlet.


b. mass merchandiser.
100.0%

c. factory outlet.


d. warehouse membership club.


e. None of the above.


Score: 0.1 / 0.1

Question 7 (0.10 points)
St. Louis-based Panera Bread LLC does not own the Panera Bread Company bakery-caf in South
Charleston. It is owned and operated by Sam Covelli of Warren, Ohio. Mr. Covelli (or, to be technically
precise, Covelli Enterprises, Inc.) has paid Panera Bread LLC an agreed-upon initial fee of a few hundred
thousand dollars, as well as paying royalties on the restaurants gross sales on a continual basis. In return,
Mr. Covelli receives the right to use Panera Bread LLCs business format, and he agrees to follow a thick
book of rules as to how the location is to be operated. In this situation, Panera Bread LLC is the __________
and Covelli Enterprises, Inc. is the __________.
Student response:
!ercent
Value
Student
Response
Answer Choices


a. producer/manufacturer; supply chain
100.0%

b. franchisor; franchisee


c. retailer; wholesaler


d. parent corporation; subsidiary


e. franchisee; franchisor


Score: 0.1 / 0.1

Question 8 (0.10 points)
Retailers control the six Ps of the retailing mix. The six Ps include the four Ps of the marketing mix PLUS:
Student response:
!ercent
Value
Student
Response
Answer Choices


a. positioning and placement.


b. potential and perception.


c. personality and persuasion.
100.0%

d. personnel and presentation.


e. payment and parsimony.


Score: 0.1 / 0.1

Question 9 (0.10 points)
Which of the following statements about a retailers promotion strategy is TRUE?
Student response:
!ercent
Value
Student
Response
Answer Choices


a. The design of the promotion strategy would
be done separately from the creation of the
retailing mix.
100.0%

b. The goal of a retail store's promotion mix is
to position the store in consumers' minds.


c. Most advertising for retailers is carried out
at the national level.


d. Retail promotion strategy usually does not
include public relations activities, such as
special events .


e. Retailers engage in very little advertising.


Score: 0.1 / 0.1

Question 10 (0.10 points)
The FIRST task of developing a retail strategy is to:
Student response:
!ercent
Value
Student
Response
Answer Choices


a. create a buying organization.


b. decide what to buy.
100.0%

c. define the target market.


d. develop the promotional strategy.


e. define the pricing policy.


Score: 0.1 / 0.1
Question 1 (0.10 points)
_____________ coordinates and integrates all the activities performed by channel members into a seamless
process, from the source to the point of consumption, resulting in enhanced customer and economic value.
Student response:
!ercent
Value
Student
Response
Answer Choices


a. Yield management


b. Channel power


c. Materials handling
100.0%

d. Supply chain management


e. The U.S. Department of Commerce


Score: 0.1 / 0.1

Question 2 (0.10 points)
Supply chain management:
Student response:
!ercent
Value
Student
Response
Answer Choices


a. is a physical flow process.


b. is customer driven.


c. provides enhanced customer value and
economic value.


d. plays the role of communicating customer
demand to channel members.
100.0%

e. is accurately described by all of the above.


Score: 0.1 / 0.1

Question 3 (0.10 points)
Which of the following is NOT a logistical component of the supply chain?
Student response:
!ercent
Value
Student
Response
Answer Choices
100.0%

a. Capital equipment financing


b. Procurement


c. Inventory control


d. Warehousing and materials handling


e. Order processing


Score: 0.1 / 0.1

Question 4 (0.10 points)
Girls can log onto www. Barbie.com and design their own special Barbie doll. They can choose the dolls skin
tone, hair color, clothes, accessories, and name. The doll is created to the new owner's specifications and
mailed to her by Mattel. With this doll, Mattel is using:
Student response:
!ercent Student Answer Choices
Value Response


a. market aggregation.


b. niche marketing.


c. a production orientation.


d. push distribution.
100.0%

e. mass customization.


Score: 0.1 / 0.1

Question 5 (0.10 points)
According to experts at a conference on supply chain management, increased traceability is the key
requirement needed to improve today's supply chains. In this instance, traceability refers to:
Student response:
!ercent
Value
Student
Response
Answer Choices


a. a carrier's ability to provide the appropriate
equipment and conditions for moving
specific kinds of goods.


b. the consistency with which a carrier delivers
goods on time.


c. a carrier's ability to move goods over a
specific route.
100.0%

d. the relative ease with which a shipment can
be located and transferred.


e. the total amount a specific carrier charges
to move the product.


Score: 0.1 / 0.1

Question 1 (.1 points)
A discrepancy of _______________ is the difference between the amount of a product produced by a
manufacturer (such as a trainload of Honey Nut Cheerios) and the amount that an individual end user wants
to buy (such as one box).
Student response:
!ercent
Value
Student
Response
Answer Choices


a. space
100.0%

b. quantity


c. assortment


d. accumulation


e. forecasting


Score: 0.1 / 0.1

Question 2 (.1 points)
McKesson Company provides health care products to pharmacies. For example, it purchases bandages,
gauze, antibacterial cream, ointments-and anything else you might need to treat a cut or scrape-from a
variety of manufacturers. It then sells to each pharmacy the combination of these items that is right for that
store. McKesson is aiding consumers by addressing a _______________.
Student response:
!ercent
Value
Student
Response
Answer Choices


a. spatial discrepancy


b. discrepancy of quantity
100.0%

c. discrepancy of assortment


d. demand discrepancy


e. discrepancy of product specifications


Score: 0.1 / 0.1

Question 3 (.1 points)
W. W. Grainger, Inc. is one of the world's largest business-to-business distributors of equipment, component
parts, and supplies in the United States and Canada. It has ownership title to over 220,000 products, which
are stocked in one national and nine regional warehouses to guarantee product availability and quick service
to the many manufacturers who are its customers. W.W. Grainger is an example of a(n):
Student response:
!ercent
Value
Student
Response
Answer Choices


a. agent or broker.
100.0%

b. merchant wholesaler.


c. retailer.


d. hands-off facilitator.


e. licensor.


Score: 0.1 / 0.1

Question 4 (.1 points)
Transactional channel functions include all of the following activities EXCEPT:
Student response:
!ercent
Value
Student
Response
Answer Choices


a. contacting buyers.


b. promoting the products to be sold.


c. taking the risks associated with product
inventories.


d. negotiating sales.
100.0%

e. physical distribution and sorting.


Score: 0.1 / 0.1

Question 5 (.1 points)
The population of Oak Hill, West Virginia is about 7,200. The population of Bluefield, West Virginia, is just
over 11,000. The population of Ripley is slightly more than 3,000. Each of these towns has one Chevrolet
dealer: King Coal Chevrolet serves Oak Hill and the surrounding trade area, Cole Chevrolet-Buick-GMC-
Cadillac serves Bluefield and the surrounding trade area, and Tate-Porter Chevrolet-Buick serves Ripley and
the surrounding trade area. From these statistics, we can conclude that the Chevrolet Division of General
Motors engages in
Student response:
!ercent
Value
Student
Response
Answer Choices


a. intensive distribution.


b. deficient distribution.


c. selective distribution.
100.0%

d. exclusive distribution.


e. disintermediation.


Score: 0.1 / 0.1
Question 1 (0.10 points)
Before buying a service, consumers may find it difficult to evaluate the 6uality of that service-especially
compared with the ease of evaluating a tangible good before purchase. This is because services usually have
fewer _________ qualities.
Student response:
!ercent
Value
Student
Response
Answer Choices


a. credence


b. subjective


c. experience
100.0%

d. search


e. relationship


Score: 0.1 / 0.1

Question 2 (0.10 points)
Virtual Bellhop is a company that ships awkward sporting goods to vacation destinations so its customers do
not have to worry about lugging them through airports or having them damaged in transit. During the
summer months and December, the company is extremely busy packaging and shipping sporting gear.
During other months that are not popular vacation months, it may only have one customer a day. This
inconsistency of demand means Virtual Bellhop has to deal with the problems associated with the service
characteristic of:
Student response:
!ercent
Value
Student
Response
Answer Choices


a. intangibility.


b. truculence.
100.0%

c. perishability.


d. heterogeneity.


e. inseparability.


Score: 0.1 / 0.1

Question 3 (0.10 points)
A continuing problem for most Minor League Baseball (MiLB) teams is low attendance. Event and venue
sponsors want to see fans in the stadiums, and the players appreciate fan support. Given this information,
what is the best pricing strategy for MiLB teams?
Student response:
!ercent
Value
Student
Response
Answer Choices
100.0%

a. Patronage oriented


b. Operations oriented


c. Revenue oriented


d. Image oriented


e. Competitive benchmarked


Score: 0.1 / 0.1

Question 4 (0.10 points)
The authors of your text use the "gap model to illustrate discrepancies that can cause problems in service
delivery. These same discrepancies can influence customer evaluations of service quality in a negative way.
Which of the following is NOT one of the gaps in the gap model?
Student response:
!ercent
Value
Student
Response
Answer Choices


a. The gap between what customers want and
what management thinks they want


b. The gap between what management thinks
customers want and the quality
specifications management develops


c. The gap between the service quality
specifications and the service that is actually
provided


d. The gap between what the company
provides and what the customer is told it
provides
100.0%

e. The gap between the level of service that is
available for a bargain price and the level of
service that is available for a premium price


Score: 0.1 / 0.1

Question 5 (0.10 points)
With regard to marketing by nonprofit organizations, with which of the following statements would the
authors of your text likely DISAGREE?
Student response:
!ercent
Value
Student
Response
Answer Choices


a. The target market of a nonprofit
organization may consist of people who do
not care about-or are even opposed to-
receiving the services that the organization
offers.


b. Nonprofit organizations often market
products or services that are by definition
"very low involvement.
100.0%

c. The marketing mix for nonprofit
organizations consists of 3 Ps: product,
place (distribution), and promotion.


d. Many nonprofit organizations are prohibited
from advertising. For these organizations,
public service announcements (PSAs) may
be an important element of the promotion
strategy,.


e. The benefits of nonprofit organizations
product offerings may be more difficult to
communicate than those of for-profit
marketers.


Score: 0.1 / 0.1
Question 1 (0.10 points)
Companies that are MOST likely to succeed in the introduction and development of new products typically:
Student response:
!ercent
Value
Student
Response
Answer Choices


a. establish an environment conducive to
achieving new-product objectives.


b. make the long-term commitment needed to
support innovation and new-product
development.


c. capitalize on experience to achieve and
maintain competitive advantage.


d. use a company-specific approach that is
driven by corporate objectives and
strategies with a well-defined new-product
strategy at its core.
100.0%

e. do all of the above.


Score: 0.1 / 0.1

Question 2 (0.10 points)
_____________ occurs when a marketer targets an existing product at new markets or segments, or if the
marketer attempts to change the current markets perception of the product.
Student response:
!ercent
Value
Student
Response
Answer Choices


a. Discontinuous innovation


b. Fusion of innovation
100.0%

c. Repositioning


d. Customer disaggregation


e. Disintermediation


Score: 0.1 / 0.1

Question 3 (0.10 points)
Which of the following statements about how adopters participate in the diffusion process is FALSE?
Student response:
!ercent
Value
Student
Response
Answer Choices


a. Early adopters may also be opinion leaders.


b. Laggards are the last consumers to adopt a
new product.
100.0%

c. There are four categories of buyers who will
eventually adopt a consumer product.


d. Skepticism is a dominant characteristic of
members of the late majority.


e. Innovators are the first to purchase a new
product.


Score: 0.1 / 0.1

Question 4 (0.10 points)
Which of these is NOT a characteristic that would slow a new products rate of adoption?
Student response:
!ercent
Value
Student
Response
Answer Choices


a. Product complexity


b. Infrequent opportunity for members of the
target market to observe the product being
used by others


c. Inability of members of the target market to
try the product
100.0%

d. High compatibility with existing knowledge
and values of members of the target market


e. No perceivable advantage over substitute
products


Score: 0.1 / 0.1

Question 5 (0.10 points)
Which of the following is CORRECT regarding the product life cycle?
Student response:
!ercent
Value
Student
Response
Answer Choices


a. A long-run drop in sales usually signals the
beginning of the maturity stage.


b. Marketers do not yet understand the
relationship between the adopter categories
and the stages of the PLC.


c. Production and promotion costs are
particularly high during the growth stage,
and the company or companies producing
the product experience(s) a loss rather than
a profit.


d. Normally, the longest stage of the product
life cycle is the introductory stage.
100.0%

e. Managements commitment of resources to
the new product can influence the length or
brevity of the introductory stage.


Score: 0.1 / 0.1

Jake Blake stopped at a concession stand in the airport terminal to buy todays Wall Street Journal. While he
was paying for his newspaper, he noticed a Sudoku puzzle book, which he also purchased. In this example,
the puzzle book is an example of a(n) ______________ product.
Student response:
!ercent
Value
Student
Response
Answer Choices


a. specialty
100.0%

b. convenience


c. business


d. heterogeneous shopping product


e. intangible


Score: 0.1 / 0.1

Question 2 (0.10 points)
A ___________ is the part of the brand that can be spoken.
Student response:
!ercent
Value
Student
Response
Answer Choices


a. brand mark


b. service mark


c. trademark
100.0%

d. brand name.


e. certification mark


Score: 0.1 / 0.1

Question 3 (0.10 points)
All other things being equal, when brand awareness is high, and the brand is perceived to be of high quality,
and customers of the brand are extremely loyal to it, the brand can be said to have __________.
Student response:
!ercent
Value
Student
Response
Answer Choices
100.0%

a. high brand equity


b. a monopoly


c. brand security


d. synonymity


e. economies of scale


Score: 0.1 / 0.1

Question 4 (0.10 points)
One out of every two ceiling fans sold in the United States is a Hampton Bay fan. Hampton Bay fans are
available only at Home Depot. This is an example of a(n) __________ brand.
Student response:
!ercent
Value
Student
Response
Answer Choices


a. manufacturers


b. national


c. generic
100.0%

d. private-label


e. gray market


Score: 0.1 / 0.1

Question 5 (0.10 points)
Veronica Lamonica bought a package of Bounce fabric sheets with Febreeze citrus scent. Since both brands
were prominently listed on the package, this would be an example of:
Student response:
!ercent
Value
Student
Response
Answer Choices


a. private-label branding.


b. complementary branding.


c. product line contraction.


d. piggyback branding.
100.0%

e. cobranding.


Score: 0.1 / 0.1

Question 1 (0.10 points)
Through marketing research, the Boston Symphony Orchestra (BSO) learned it has an older market and is
not attracting younger concertgoers. It next conducted marketing research to determine if an integrated
advertising campaign targeted to the younger market would be successful. In its second use of research,
BSO employed _____ marketing research.
Student response:
!ercent
Value
Student
Response
Answer Choices


a. normative


b. subjective


c. descriptive
100.0%

d. predictive


e. historical


Score: 0.1 / 0.1

Question 2 (0.10 points)
_____ is the process of planning, collecting, and analyzing data relevant to marketing decision making. The
results of this analysis are then communicated to management.
Student response:
!ercent
Value
Student
Response
Answer Choices


a. Artificial intelligence


b. Decision framing


c. A feasibility study
100.0%

d. Marketing research


e. Decision support


Score: 0.1 / 0.1

Question 3 (0.10 points)
Data previously collected for purposes other than the one at hand may be an important source of
information as the researcher goes about defining the problem. These data are called ___________ data.
Student response:
!ercent Student Answer Choices
Value Response


a. consensual


b. primary


c. single-source


d. subjective
100.0%

e. secondary


Score: 0.1 / 0.1

Question 4 (0.10 points)
An engineer at Volkswagen America spent three and a half hours on a Greyhound bus to observe the vast
distances that Americans travel in their automobiles and to try to understand why Americans treat their
automobiles as rolling extensions of their living rooms. What kind of data did the engineer collect?
Student response:
!ercent
Value
Student
Response
Answer Choices


a. heuristic
100.0%

b. primary


c. cohesive


d. compromised


e. secondary


Score: 0.1 / 0.1

Question 5 (0.10 points)
Ballistic Bazaar has outgrown its building. Its owner wants to know how its customers will react to a move
and if the addition of new products and services will make the new location more appealing to its loyal
clientele. The owner has surveyed several customers to gauge their reaction to various potential locations as
well as their reactions to various prospective service mixes. The survey results are ______________ data.
Student response:
!ercent
Value
Student
Response
Answer Choices


a. observation


b. secondary


c. collaborative
100.0%

d. primary


e. posthumous


Score: 0.1 / 0.1

Question 6 (0.10 points)
What is the chief advantage of primary data over secondary data?
Student response:
!ercent
Value
Student
Response
Answer Choices


a. Primary data can usually be obtained for a
lower cost.
100.0%

b. Primary data are likely to be more relevant
to the problem at hand.


c. Primary data are available for any interested
party to see and use.


d. Primary data can often be obtained from a
public library.


e. Primary data are immune from
measurement error and sampling error.


Score: 0.1 / 0.1

Question 7 (0.10 points)
The most popular method for gathering primary data is ______________, in which the researcher interacts
with people to obtain facts, opinions, and attitudes.
Student response:
!ercent
Value
Student
Response
Answer Choices
100.0%

a. survey research


b. simple observation


c. experimental research


d. database mining


e. None of the above.


Score: 0.1 / 0.1

Question 8 (0.10 points)
As a marketing research method, a major limitation of the mall intercept interview (where researchers send
inteviewers to approach consumers at a mall and ask them a planned set of questions) is
Student response:
!ercent
Value
Student
Response
Answer Choices


a. the interviewers inability to probe or ask for
clarification.


b. the requirement that people be interviewed
while theyre in a walking or standing
position.


c. its high cost, compared to an in-home
interview.


d. the difficulty of using a new-product
demonstration as part of the research
process.
100.0%

e. the difficulty of obtaining a truly random
sample of the population that is of interest.


Score: 0.1 / 0.1

Question 9 (0.10 points)
Low response rate is a problem commonly associated with which marketing research method?
Student response:
!ercent
Value
Student
Response
Answer Choices


a. Focus groups


b. Executive interviews
100.0%

c. Mail surveys


d. Mall intercept interviews


e. In-home interviews


Score: 0.1 / 0.1

Question 10 (0.10 points)
Which of the following do the authors of the textbook NOT present as an advantage of Internet surveys,
compared to other, more traditional survey methods?
Student response:
!ercent
Value
Student
Response
Answer Choices
100.0%

a. They are the best way to reach individuals
who are over 65, adults who have less than
a high school education, and the lowest-
income consumer segments.


b. They result in higher respondent
participation rates.


c. They enable managers to have faster access
to research data.


d. They allow researchers to personalize
questions to individual respondents.


e. They give researchers access to hard-to-
reach respondents, such as high-income
professionals and top managers.


Score: 0.1 / 0.1
Question 1 (0.10 points)
The marketing researcher for Chickadee, a manufacturer of designer clothing for infants and toddlers, has
submitted a report on potential segments for expansion of the companys marketing efforts. She noted an
increase in the number of professional women over the age of 35 who are leaving their jobs to have their
first baby. In her report, she described this segment as "having substantiality. This means that the "Over-
35, First-Time Mom segment:
Student response:
!ercent
Value
Student
Response
Answer Choices


a. has enough of its own special stores,
magazines, and other outlets that it will be
possible to direct advertising at this group.
100.0%

b. is a large enough segment to be profitable
for the marketer that develops a marketing
mix for it.


c. can be expected to respond differently to a
marketing mix than other segments might
respond.


d. is too large to be targeted effectively, and
needs to be reduced to a more easily
identifiable and measurable size.


e. will be difficult to satisfy with a marketing
mix.


Score: 0.1 / 0.1

Question 2 (0.10 points)
In the early years of the 21
st
century, Ford Motor Company used the slogan "No Boundaries for its ad
campaigns for Ford SUVs. It invited people who wanted ultimate people-carrying and gear-storing
capabilities "to be outfitted with the most far-reaching sports utility vehicle on earth. If the SUV campaign
had been simply geared to families with young children, Ford would have been using __________
segmentation. However, because the campaign was trying to appeal to people who don't like the restrictions
of a sedan, who want to be able to pack up and leave at a moment's notice, and who enjoy freedom and
independence, Ford was relying on __________ segmentation.
Student response:
!ercent
Value
Student
Response
Answer Choices


a. geographic; demographic


b. psychographic; usage rate


c. benefit; demographic


d. usage rate; demographic
100.0%

e. demographic; psychographic


Score: 0.1 / 0.1

Question 3 (0.10 points)
Shirley Turley owns Ballistic Bazaar (a gun shop and shooting range). She decides she can segment her
market into three distinct groups: those who want a weapon for home protection, those who want a weapon
for seasonal hunting of small game (a.k.a. `varmints), and collectors who enjoy owning a wide variety of
firearms for their intrinsic and aesthetic value. Thus, she segments her market into those who are seeking
safety for their household and/or family, those who are seeking recreation, and those who are seeking the
satisfaction of ownership in and of itself. Which label most closely resembles the dimension Ms. Turley is
using to segment her market?
Student response:
!ercent
Value
Student
Response
Answer Choices


a. Geographic segmentation
100.0%

b. Benefit segmentation


c. Income segmentation


d. Family life-cycle (FLC) segmentation


e. Age segmentation


Score: 0.1 / 0.1

Question 4 (0.10 points)
Sam and Phil Barshop developed the La Quinta Inn chain of hotels with the aim of satisfying the lodging
needs of a specific type of traveler: the cost-conscious white-collar business traveler. By focusing upon this
market segment, the Barshop brothers were practicing _____________ targeting.
Student response:
!ercent
Value
Student
Response
Answer Choices


a. undifferentiated


b. multisegment
100.0%

c. concentrated


d. aggregated


e. one-to-one


Score: 0.1 / 0.1

Question 5 (0.10 points)
What does the term "product positioning refer to?
Student response:
!ercent
Value
Student
Response
Answer Choices


a. Efforts by marketers to have their brands
used by high-visibility characters in movies
and TV shows


b. Efforts by marketers to secure the best shelf
locations in major chain supermarkets and
drugstores


c. Geographic segmentation, often within
major metropolitan areas
100.0%

d. The place a product occupies in consumers
minds-in comparison with competitors
products-based on important features


e. The ranking of a product in Interbrands (a
global consulting group) annual "100 Best
Global Brands. For example, Coca-Cola is
ranked #1 in the 2010 "100 Best Global
Brands and Hewlett-Packard is ranked #10)


Score: 0.1 / 0.1


Question 1 (0.10 points)
When Caterpillar, the manufacturer of earthmoving equipment, builds a piece of equipment, it may make
use of several items that were produced by other companies. Thus, the finished Caterpillar product may
have a drivetrain (engine-transmission-axle combination), a set of electronic controls, a cooling system, and
an undercarriage track system that have all been manufactured by a company other than Caterpillar. As a
business customer for the products that become part of its own finished products, Caterpillar can best be
categorized as:
Student response:
!ercent
Value
Student
Response
Answer Choices


a. a reseller.


b. an institution.
100.0%

c. an original equipment manufacturer (OEM).


d. a government organization.


e. None of the above.


Score: 0.1 / 0.1

Question 2 (0.10 points)
Which of the following does NOT describe the business market, as compared to the consumer market?
Student response:
!ercent
Value
Student
Response
Answer Choices
100.0%

a. Organizational buying decisions are usually
made independently by a purchasing agent,
while consumer buying decisions are made
jointly.


b. Business customers tend to be more
geographically concentrated, and customers
in consumer markets tend to be more
geographically dispersed.


c. Business markets are served by a direct
channel of distribution than are consumer
markets.


d. Purchasing by businesses is a more formal
process than purchasing by consumers.


e. Business marketers tend to have far fewer
customers than consumer marketers.


Score: 0.1 / 0.1

Question 3 (0.10 points)
As a result of ____________, a small increase or decrease in consumer demand can produce a much larger
change in demand for the facilities and equipment needed to manufacture the consumer product.
Student response:
!ercent
Value
Student
Response
Answer Choices


a. the demand fluctuation principle


b. the joint demand principle


c. inelastic demand
100.0%

d. the multiplier effect


e. legal constraints


Score: 0.1 / 0.1

Question 4 (0.10 points)
The buying center:
Student response:
!ercent
Value
Student
Response
Answer Choices


a. will be made up of the same organizational
members for as long as the purchasing
process lasts.


b. is typically diagrammed on a companys
organization chart and usually reports to the
vice president for operations.


c. is made up of people who are all employed
in the purchasing department.
100.0%

d. refers to all members of an organization
who become involved in the buying process
for a particular product or service.


e. has its own building, in most organizations.


Score: 0.1 / 0.1

Question 5 (0.10 points)
Madea Padilla works as a salesperson for a company that provides customized motivational seminars to
medium-sized businesses. When contacting a potential client, she often talks to several people before she
locates someone who can give her a purchase order or a refusal. In terms of buying center roles, Madea is
having trouble identifying the ___________.
Student response:
!ercent
Value
Student
Response
Answer Choices


a. influencer


b. gatekeeper


c. user
100.0%

d. decider.


e. purchaser.


Score: 0.1 / 0.1

Question 1 (0.10 points)
___________ is "the amount of time and effort a buyer invests in the search, evaluation, and decision
processes of consumer behavior.
Student response:
!ercent
Value
Student
Response
Answer Choices


a. Sunk cost
100.0%

b. Involvement


c. Opportunity cost


d. Temporal cost


e. Perceived risk


Score: 0.1 / 0.1

Question 2 (0.10 points)
All of the following factors directly influence consumers' level of involvement in the purchase process
EXCEPT:
Student response:
!ercent
Value
Student
Response
Answer Choices
100.0%

a. the consumers level of education.


b. the consumers previous experience with the
type of product.


c. the financial risk associated with the
purchase.


d. the social visibility of the purchased item.


e. consumers level of ongoing interest in the
product category.


Score: 0.1 / 0.1

Question 3 (0.10 points)
Harley Farley has little to no preference for a brand of tires. He says, "Goodyear, Bridgestone, Toyo,
Michelin, Uniroyal, Dunlop, BF Goodrich, Cooper, Hankook, Yokohama--they're all about the same. Round,
black, and made of rubber." Harley will go only one place to buy tires for his truck and for the family car:
National Tire and Battery. If one of the vehicles needs tires, Harley will go to the NTB store. Once he's there,
he'll decide which brand to purchase based on price and warranty. Harleys tire shopping behavior most
closely resembles:
Student response:
!ercent
Value
Student
Response
Answer Choices


a. decelerated decision making.


b. routine response behavior.
100.0%

c. limited decision making.


d. extensive decision making.


e. tertiary decision making.


Score: 0.1 / 0.1

Question 4 (0.10 points)
In the United States, social class (for example, "working class, "upper middle class)
Student response:
!ercent
Value
Student
Response
Answer Choices
100.0%

a. can be a useful distinction to marketers-it
helps them identify and understand lifestyle
differences between groups.


b. is the same as income level.


c. is too complicated an issue to be useful to
marketers.


d. is not relevant; the U.S. is a classless
society.


e. is not a very useful distinction to marketers
because all social classes shop at the same
stores.


Score: 0.1 / 0.1

Question 5 (0.10 points)
Which of the following is a cultural influence on consumer buying decisions?
Student response:
!ercent
Value
Student
Response
Answer Choices


a. Reference groups


b. Perception


c. Opinion leaders


d. Stage in the family life cycle
100.0%

e. Social class


Score: 0.1 / 0.1

Question 6 (0.10 points)
A few years ago, the Australian footwear and accessories brand UGG was an unknown brand in the United
States. A few tabloid shots of actresses like Kate Hudson and Cameron Diaz parading around in their UGG
boots were instrumental in making the brand popular in the U.S. market; sales havent let up since. Diaz
and Hudson more than likely acted as:
Student response:
!ercent
Value
Student
Response
Answer Choices
100.0%

a. opinion leaders.


b. laggards.


c. inducers of cognitive dissonance.


d. gatekeepers.


e. initiators.


Score: 0.1 / 0.1

Question 7 (0.10 points)
Social influences on consumer buying decisions include:
Student response:
!ercent
Value
Student
Response
Answer Choices


a. societal values, culture, and family.


b. reference groups, societal values, opinion
leaders, and family.


c. personality, lifestyle, and reference groups.
100.0%

d. reference groups, opinion leaders, and
family.


e. lifestyle, reference groups, and family.


Score: 0.1 / 0.1

Question 8 (0.10 points)
Individual influences on consumer buying decisions include:
Student response:
!ercent
Value
Student
Response
Answer Choices


a. society, culture, and family.


b. reference groups, societal values, opinion
leaders, and family.
100.0%

c. stage in family life cycle, personality, and
gender.


d. perception, motivation, and subculture.


e. lifestyle, reference groups, and family.


Score: 0.1 / 0.1

Question 9 (0.10 points)
Self-concept is how consumers perceive themselves in terms of __________, __________, __________,
and __________.
Student response:
!ercent
Value
Student
Response
Answer Choices


a. socialization; gender; opinion leaders;
societal values


b. subculture; reference groups; age; learning


c. motivation; subculture; gender; family


d. stage in family life cycle; socialization;
reference groups; subculture
100.0%

e. attitudes; perceptions; beliefs; self-
evaluations


Score: 0.1 / 0.1

Question 10 (0.10 points)
Psychological influences on consumer buying decisions include:
Student response:
!ercent
Value
Student
Response
Answer Choices


a. society, culture, and family.


b. reference groups, societal values, and
opinion leaders.


c. stage in family life cycle, lifestyle, and
gender.
100.0%

d. perception, motivation, and learning.


e. age, gender, and self-concept.


Score: 0.1 / 0.1
Question 1 (0.10 points)
Which of the following statements about multinational firms is TRUE?
Student response:
!ercent
Value
Student
Response
Answer Choices


a. Multinationals main involvement in
international trade is limited to importing
and exporting.


b. Most multinational corporations borrow
money from the World Bank to finance their
first international business ventures.


c. To fit the description of a multinational
corporation, a company must be involved in
either joint venturing or direct investment in
at least one country outside the country in
which it is headquartered.


d. All of the above.
100.0%

e. None of the above.


Score: 0.1 / 0.1

Question 2 (0.10 points)
With a __________, a firm produces standardized products that will be sold the same way all over the
world, as Ford Motor Company does with the Focus automobile.
Student response:
!ercent
Value
Student
Response
Answer Choices


a. traditional marketing strategy
100.0%

b. global marketing standardization approach


c. product adaptation approach


d. dumping


e. None of the above.


Score: 0.1 / 0.1

Question 3 (0.10 points)
_____________ is a legal process whereby a firm agrees to let another firm use its manufacturing process,
trademarks, patents, trade secrets, or other proprietary knowledge in return for a fee or royalty.
Student response:
!ercent
Value
Student
Response
Answer Choices


a. A joint venture


b. Divestment


c. Counterfeiting
100.0%

d. Licensing


e. Contract manufacturing


Score: 0.1 / 0.1

Question 4 (0.10 points)
Delta Galil Industries Ltd is a clothing manufacturer based in Israel. It is responsible for making clothing sold
under the Playtex brand, Donna Karan/DKNY brand, Hugo Boss brand, and Calvin Klein brand. When one
company manufactures clothing that is sold using another companys brand, _____ has occurred.
Student response:
!ercent
Value
Student
Response
Answer Choices


a. counterfeiting


b. joint venturing


c. crossdocking
100.0%

d. contract manufacturing


e. direct investment


Score: 0.1 / 0.1

Question 5 (0.10 points)
Which method of entering the global marketplace involves the GREATEST financial commitment and risk?
Student response:
!ercent
Value
Student
Response
Answer Choices


a. Exporting


b. Licensing


c. Contract manufacturing


d. A joint venture
100.0%

e. Direct investment


Score: 0.1 / 0.1
Question 1 (0.10 points)
When planning marketing strategies during times of inflation, marketers need to be aware that inflation
causes consumers to:
Student response:
!ercent
Value
Student
Response
Answer Choices


a. purchase more goods and services to
support their psychological selves, such as
counseling and stress management training.


b. consume more meals away from home.


c. buy more luxuries and non-necessities than
in non-inflationary times.
100.0%

d. not pay more for a product than the
subjective value that they place on it.


e. put more money into savings accounts
because prices are too high.


Score: 0.1 / 0.1

Question 2 (0.10 points)
Marketing managers cannot control ___________, but they can influence it.
Student response:
!ercent
Value
Student
Response
Answer Choices


a. where advertising is placed
100.0%

b. the external environment


c. the sales force


d. the distribution strategy


e. how products are priced


Score: 0.1 / 0.1

Question 3 (0.10 points)
Zimmer Centralpulse is the world's largest producer of replacement hips and knees. It is particularly
interested in dominating the market in the southwestern United States, where a large aging population lives.
Which external environmental element most directly influences the company's growth?
Student response:
!ercent
Value
Student
Response
Answer Choices


a. Economic conditions


b. Political and legal factors


c. Cultural beliefs and values
100.0%

d. Demography


e. Social forces


Score: 0.1 / 0.1

Question 4 (0.10 points)
According to the authors of your textbook, which of the following events has had a greater effect on
marketing than any other social change?
Student response:
!ercent
Value
Student
Response
Answer Choices


a. The growing number of one-child families


b. The increasing number of immigrants from
Northern Europe


c. The growing number of people who make
frequent use of the Internet
100.0%

d. The increasing number of working women


e. The growing number of people who are
continuing to work past normal retirement
age


Score: 0.1 / 0.1

Question 5 (0.10 points)
This month, Dean Sheen is opening a new workout facility and full-service day spa. Dean knows that the
success of his business (which he has named "The Sweat, Stink & Suffer Health Spa, also known as "The
Triple S) depends largely upon the following three environmental conditions:


--Physical fitness has to be a high priority for an increasing number of members of the public.


--Modern exercise equipment is computerized. Hugh will have to learn how to program some of it or employ
somebody with that expertise.


--Hell have to quickly learn and abide by the many laws that apply to running a for-profit service
establishment, such as worker and customer safety regulations and federal, state, and local tax
requirements.


Of the six external environmental factors that are presented in your text, which three appear to have the
greatest impact on Deans business? That is, which three are illustrated by the above issues?
Student response:
!ercent
Value
Student
Response
Answer Choices


a. Demographic, competitive, and economic


b. Political/legal, technological, and
demographic


c. Competitive, social, and technological


d. Social, political/legal, and demographic
100.0%

e. Social, technological, and political/legal.


Score: 0.1 / 0.1
Question 1 (0.10 points)
Ethical development can be thought as having three levels. The most basic, childlike level is the _____
stage.
Student response:
!ercent
Value
Student
Response
Answer Choices
100.0%

a. preconventional morality


b. transconventional morality


c. conventional morality


d. postconventional morality


e. panconventional morality


Score: 0.1 / 0.1

Question 2 (0.10 points)
Kiefer Schieffer is a sales manager at Arrow Electronics. Kiefer makes sure that the salespeople under his
supervision receive their commission checks on a timely basis; he does this because he knows that if
salespeople become disgruntled over matters such as late commission checks, they are likely to seek
employment elsewhere. Kiefer doesnt want high employee turnover on his own managerial record, nor does
he want Arrow Electronics to develop a reputation as a lousy place to be employed as a salesperson-that
would make the recruiting part of his job more difficult than he wants it to be. Kiefer also reminds his
salespeople not to outright lie to customers-or even stretch the truth-to make a sale; he knows that if a
customer can prove in a court of law that the company has been dishonest, it could cost Arrow Electronics a
lot of money and could also cost Kiefer his job. Kiefer Schieffer is most likely at the ________ stage of
ethical development.
Student response:
!ercent
Value
Student
Response
Answer Choices


a. preconventional morality


b. primordial morality
100.0%

c. conventional morality


d. postconventional morality


e. postmodern enlightened


Score: 0.1 / 0.1

Question 3 (0.10 points)
Bryce Price is a new-vehicle salesman at Diller Chevrolet. Bryce is very concerned about treating each
customer fairly and developing long-term relationships with them. He will not try to sell a customer a vehicle
that the customer clearly cannot afford, and he contacts each customer personally within two weeks after a
purchase to say "Thank you and to make sure the vehicle is meeting the customers expectations. If the
customer mentions a problem with the vehicle during Bryces follow-up call, Bryce will contact Dillers service
department to make an appointment for the customer, rather than simply suggesting that the customer call
the service department. On more than one occasion, Bryce has had his wife drop him at an elderly
customers home so that he could drive the customers vehicle to the dealerships service department
himself. He told your professor: "Its pretty simple, really. Thats what I would want a car salesman to do for
my grandmother if she bought a car from him and there was a problem. Bryce Price is most likely at the
______________ stage of ethical development.
Student response:
!ercent
Value
Student
Response
Answer Choices


a. preconventional morality


b. primordial morality


c. conventional morality
100.0%

d. postconventional morality


e. compromised morality


Score: 0.1 / 0.1

Question 4 (0.10 points)
Eli Lilly and Company is an Indianapolis-based pharmaceutical company whose medicines are marketed in
143 countries. Its well-known products include Prozac, Zyprexa, Cymbalta, and Cialis, among others. On
August 8, 2010, The Chronicle of Philanthropy reported that Eli Lilly and Companys charitable gifts of cash
and product totaled $405,633,000 in 2009. The company is also known for encouraging and facilitating its
employees commitment to volunteerism and community service through innovative programs such as its
"Global Day of Service, "Lilly Science Education Outreach (LSEO), and "The Lilly MDR-TB Partnership.
These efforts and activities by Eli Lilly and Company are an example of a corporations commitment to
Student response:
!ercent
Value
Student
Response
Answer Choices


a. obeying the law.


b. managing its excess inventory.


c. generating projects to keep its employees
busy.
100.0%

d. fulfilling its corporate social responsibility.


e. distracting its critics.


Score: 0.1 / 0.1

Question 5 (0.10 points)
"My responsibility as an executive at this company is to apply my business skills to the best of my ability, in
order to make the most profit possible for the owners of the company-who have entrusted me with the
responsibility to do so. I also have an obligation to make sure that neither I nor any of my employees break
any laws as we go about the business of the company. As an executive at this company, it is not my
responsibility to go above and beyond what the law requires and try to solve the problems of society. I am a
businessperson. I have the training and the abilities to run this business. Social workers and community
development specialists are the people with the knowledge and abilities to solve societys problems. Its best
to leave that job to them, and its best for the social workers and community development specialists to
leave the job of running this company to me. I and my employees are free to contribute to social and
charitable causes on our own time and of our own resources, if we so choose. A high-ranking manager who
would make this statement believes that CSR (corporate social responsibility) consists of __________.
Student response:
!ercent
Value
Student
Response
Answer Choices


a. economic responsibility only


b. philanthropic responsibility only
100.0%

c. economic responsibility and legal
responsibility


d. legal responsibility and ethical responsibility


e. economic responsibility, legal responsibility,
ethical responsibility, and philanthropic
responsibility


Score: 0.1 / 0.1
Question 1 (0.10 points)
The authors of your text define "__________ as "the managerial process of creating and maintaining a fit
between the organizations objectives and resources and the evolving market opportunities.
Student response:
!ercent
Value
Student
Response
Answer Choices


a. marketing


b. relationship marketing


c. environmental scanning


d. MOA
100.0%

e. strategic planning


Score: 0.1 / 0.1

Question 2 (0.10 points)
Which of the following statements about a marketing plan is TRUE?
Student response:
!ercent
Value
Student
Response
Answer Choices


a. Marketing plans are often passed down
through generations of managers as oral
traditions, much like recipes and
"conventional wisdom are passed down
through generations of a family.


b. A marketing plan and a marketing mix are
the same thing.


c. All marketing plans have the same format,
that is, the same sections and features, in
the same order of presentation. Ford Motor
Companys marketing plan for the 2012
Ford Fusion Hybrid, for example, can be
expected to cover the same details-in the
same order-as, say, Apple Inc.s marketing
plan for the iPhone 5.
100.0%

d. A marketing plan is created so an
organization can meet its marketing
objectives.


e. All of the above are true.


Score: 0.1 / 0.1

Question 3 (0.10 points)
Which of the following mission statements might the authors of your text say is too narrow?
Student response:
!ercent
Value
Student
Response
Answer Choices


a. "To simply delight you . . . every day (Sara
Lee)


b. "To help people and institutions prosper
(SunTrust Banks, Inc.)
100.0%

c. "To be a pre-eminent global producer and
merchant of iron ore and other steel-related
products (Cliffs Natural Resources)


d. "To create the most enjoyable shopping
experience for our guests (The Buckle,
Inc.)


e. "To maximize the long-term value of Cintas
for its shareholders and working partners by
exceeding our customers expectations
(Cintas Corporation)


Score: 0.1 / 0.1

Question 4 (0.10 points)
Consultants at Sandelman & Associates, a foodservice market research firm, are presenting a SWOT analysis
to top managers at CKE Restaurants, Inc. CKE is the parent company of the Carls Jr., Hardees, Red Burrito,
and Green Burrito chains. The Sandelman consultants mention that of all QSR (quick-service restaurant)
chains in the United States, McDonalds has the largest share of drive-through sales, followed (in this order)
by Wendys, Taco Bell, Burger King, and Sonic. For management at CKE, the fact that their own drive-
through traffic volume is not sufficient to rank in the top five would be classified as a(n)
Student response:
!ercent
Value
Student
Response
Answer Choices


a. strength.
100.0%

b. weakness.


c. opportunity.


d. threat.


e. value proposition.


Score: 0.1 / 0.1

Question 5 (0.10 points)
____________________ isnt a marketing objective, exactly.
Student response:
!ercent
Value
Student
Response
Answer Choices


a. "To improve customer satisfaction to ratings
from the December, 2010 level of 88% to
95% by December, 2012


b. "To increase ROI by 20 percent above March
31, 2011 by the end of the next fiscal year
100.0%

c. "To be the undisputed leader in our
industry


d. "To reduce the cost of discarded scrap metal
from the present level of 6% of our
manufacturing cost to less than 1% of our
total manufacturing cost by June 30, 2012


e. "To grant five additional distributorships in
the mid-Atlantic region by December 31,
2011.


Score: 0.1 / 0.1

Question 6 (0.10 points)
Arizona Tea is marketed by Ferolito Vultaggio & Sons. Ferolito Vultaggio & Sons took a basic drink and put it
into unusual bottles with elaborate designs. The wide-mouthed, long-necked bottles are now considered to
be trendsetters in the new age beverage industry, and consumers often buy the tea just for the bottle. The
success of Arizona Tea is based on:
Student response:
!ercent
Value
Student
Response
Answer Choices


a. cost competitive advantage.


b. efficient labor.


c. experience curve.


d. niche competitive advantage.
100.0%

e. product differentiation competitive
advantage.


Score: 0.1 / 0.1

Question 7 (0.10 points)
Calicchio Musical Instruments, Inc. makes expensive, handcrafted trumpets that are the preferred brand of
some of the worlds most renowned classical and jazz horn players. By targeting and serving a single target
market (i.e., only those who can afford and appreciate the product, and who use it for performing and
recording), Calicchio has achieved
Student response:
!ercent
Value
Student
Response
Answer Choices


a. multisegment targeting.
100.0%

b. a niche competitive advantage.


c. a product differentiation competitive
advantage.


d. a cost competitive advantage.


e. derived demand.


Score: 0.1 / 0.1

Question 8 (0.10 points)
The J.M. Smucker Company has been known for its "Smuckers line of jams, jellies, and preserves since the
1920s. In 2001, Smucker bought the Jif peanut butter brand from Procter & Gamble to manufacture and
market as its own. The Smucker Companys acquisition of Jif can best be categorized as a ___________
strategy.
Student response:
!ercent
Value
Student
Response
Answer Choices


a. market development


b. market penetration


c. product penetration
100.0%

d. product development


e. diversification


Score: 0.1 / 0.1

Question 9 (0.10 points)
Making sure products and services are available when and where customers want them is the job of which
element of the marketing mix?
Student response:
!ercent
Value
Student
Response
Answer Choices


a. Advertising strategies


b. Production strategies


c. Product strategies


d. Promotion strategies
100.0%

e. Distribution strategies


Score: 0.1 / 0.1

Question 10 (0.10 points)
Some Fortune 500 firms spend more than $100,000 for each salespersons training. Xerox, IBM, and Pfizer
are examples of such companies. Thats a lot of money to spend on an employee who may not successfully
complete the training program or who may leave the company after a couple of years. In marketing terms,
the label for what salespeople do on the job is "personal selling. In terms of the marketing mix, personal
selling is part of which element?
Student response:
!ercent
Value
Student
Response
Answer Choices


a. Product
100.0%

b. Promotion


c. Procurement


d. Pricing


e. Distribution


Score: 0.1 / 0.1

Question 1 (0.10 points)
The current "official American Marketing Association (AMA) definition of marketing, as quoted by the
authors of your text, is:
Student response:
!ercent
Value
Student
Response
Answer Choices
100.0%

a. " . . . the activity, set of institutions, and
processes for creating, communicating,
delivering, and exchanging offerings that
have value for customers, clients, partners,
and society at large.


b. the process of planning and executing
conception, pricing, promotion, and
distribution of goods, ideas, and services to
create exchanges that satisfy individual and
organizational goals.


c. the business activities involved in the flow of
goods and services from production to
consumption.


d. an organizational function and a set of
processes for creating, communicating, and
delivering value to customers and for
managing customer relationships in ways
that benefit the organization and its
stakeholders.


e. the performance of business activities
directed toward, and incident to, the flow of
goods and services from producer to
consumer or user.


Score: 0.1 / 0.1

Question 2 (0.10 points)
"What we think we produce is not of primary importance to our companys success. What customers think
they are buying-that is, perceived value-defines who we are. A marketing executive who makes this
statement probably works for a company that has a(n) "_______________ orientation.
Student response:
!ercent
Value
Student
Response
Answer Choices


a. production
100.0%

b. market


c. retail


d. sales


e. enterprise


Score: 0.1 / 0.1

Question 3 (0.10 points)
The "marketing concept is a philosophy that stresses that the social and economic justification for an
organization's existence is the satisfaction of customer wants and needs while:
Student response:
!ercent
Value
Student
Response
Answer Choices


a. producing a good or service at the lowest
possible cost.


b. improving the general standard of living in a
community, region, or nation.


c. constantly increasing sales volumes.


d. applying scientific management techniques
to increase efficiency.
100.0%

e. simultaneously meeting the organization's
objectives.


Score: 0.1 / 0.1

Question 4 (0.10 points)
On page 6 of your textbook, the authors state, "Firms [meaning "companies] that adopt and implement the
marketing concept are said to be market oriented, meaning they assume that a sale does not depend on
an aggressive sales force but rather on a customers decision to purchase a product. And, on page 5, the
authors say, "Sales-oriented companies often find that, despite the quality of their sales force, they cannot
convince people to buy products or services that are neither wanted nor needed. When the authors say
"sales force, to what are they referring?
Student response:
!ercent
Value
Student
Response
Answer Choices


a. The level of "forcefulness in the way the
companys advertisements are worded


b. The "pushiness of the companys
employees when they deal with customers
100.0%

c. The salespeople who are employed by the
company


d. All of the above


e. None of the above


Score: 0.1 / 0.1

Question 5 (0.10 points)
Reid Meade-an accounting major-is complaining to his friend and classmate, Molly Pauley (who is a
marketing major), about his principles of marketing class. Reid says to Molly, "I plan to go to work in a CPA
firm. Ill be doing financial statements, tax documents, and other money-related work for the firms clients.
Im not planning to be an advertising copywriter, a marketing research director, a sales manager, or a new-
product designer. Why should I care about marketing??? What should Molly say to Reid?
Student response:
!ercent
Value
Student
Response
Answer Choices


a. "If you work in a prestigious CPA firm, your
pay raises and promotions will partially
depend on how good you are at recruiting
new clients for the firm. Youll be better at
recruiting if you understand marketing.


b. "The better you understand marketing, the
wiser you will be as a consumer. Youll
spend your own money more effectively for
the rest of your life. The money you save
may amount to millions of dollars, and youll
probably gain a lot more satisfaction from
your purchases, too.


c. "Todays best-run businesses-including CPA
firms-expect every employee to strive for
maximum customer satisfaction, and
customer satisfaction is a marketing issue.
You will need to understand marketing at
every phase of your career, regardless of
what your job title is going to be.
100.0%

d. All of the above


e. None of the above


Score: 0.1 / 0.1


Question 1 (2.00 points)
An organization that has a __________ believes that it exists not only to satisfy customer wants and needs
and to meet organizational objectives, but also to preserve or enhance individuals' and society's long-term
best interests.
Student response:
!ercent
Value
Correct
Response
Student
Response
Answer Choices


a. sales orientation


b. market orientation


c. mission statement


d. production orientation
100.0%

e. societal marketing orientation


Score: 2 / 2

Question 2 (2.00 points)
A ___________ is the market segment or segments upon which a marketer decides to focus its efforts. It is
also described by the authors of the text as "a defined group that managers feel is most likely to buy a
firms product.
Student response:
!ercent
Value
Correct
Response
Student
Response
Answer Choices
100.0%

a. target market


b. demographic sample


c. aggregated user group


d. buying cartel


e. consumer cluster


Score: 2 / 2

Question 3 (2.00 points)
With regard to a marketing plan, with which of the following statements would the authors of your text most
likely AGREE?
Student response:
!ercent
Value
Correct
Response
Student
Response
Answer Choices


a. A marketing plan need not be
articulated in writing, as long as
an organization has one. In fact,
sensitive and confidential parts
of a marketing plan may be
safer from competitors
corporate spies if the plan exists
only in the minds of key
executives.


b. The elements of a marketing
plan need to be decided upon in
a "linear fashion-that is, as a
series of sequential planning
steps. None of them can be
decided upon simultaneously.


c. A marketing plan articulates
only the "creative details, such
as product design and
advertising copy. It does not
contain "time- and money-
related details, such as profit
projections and budgets.


d. All of the above
100.0%

e. None of the above


Score: 2 / 2

Question 4 (2.00 points)
Frito-Lay is considered the king of the salty snack industry. Its massive distribution network includes 42
production plants, 12,800 delivery people, and 900+ tractor-trailer rigs. Smaller manufacturers of salty
snacks are wise to monitor the activities of Frito-Lay because it is a formidable _____________ factor in
their external environment.
Student response:
!ercent Correct Student Answer Choices
Value Response Response


a. legal/political


b. technological
100.0%

c. competitive


d. social


e. demographic


Score: 2 / 2

Question 5 (2.00 points)
Dwight Knight is a calculating, self-centered salesperson. He never engages in relationship marketing. He
looks on any sales situation as a "win-lose" situation, and he believes any method he can use to close the
deal is justified by the outcome. Dwight is most likely at the __________ stage of ethical development.
Student response:
!ercent
Value
Correct
Response
Student
Response
Answer Choices
100.0%

a. preconventional morality


b. auspicious idealist


c. conventional morality


d. postmodern enlightened


e. postconventional morality


Score: 2 / 2

Question 6 (2.00 points)
Marketing is the activity, set of institutions, and processes for ___________ offerings that have value for
customers, clients, partners, and society at large.
Student response:
!ercent
Value
Correct
Response
Student
Response
Answer Choices


a. pushing


b. exaggerating


c. manipulating


d. imitating
100.0%

e. None of the above.


Score: 2 / 2

Question 7 (2.00 points)
In a SWOT analysis, the "O" stands for
Student response:
!ercent
Value
Correct
Response
Student
Response
Answer Choices


a. optimal.


b. organization.
100.0%

c. opportunities.


d. operations.


e. objectives.


Score: 2 / 2

Question 8 (2.00 points)
__________ is "the relationship between benefits and the sacrifice necessary to obtain those benefits.
Student response:
!ercent
Value
Correct
Response
Student
Response
Answer Choices


a. Product warranty


b. Customer satisfaction


c. Customer loyalty


d. Sustainability
100.0%

e. Customer value


Score: 2 / 2

Question 9 (2.00 points)
Strategic planning is "the managerial process of creating and maintaining a fit between the evolving market
opportunities and the _________.
Student response:
!ercent
Value
Correct
Response
Student
Response
Answer Choices


a. evolving demographic
characteristics of the target
market


b. organizations past and present
marketing actions


c. organizations mission statement


d. evolving knowledge and abilities
of the organizations top
executives, such as the CEO and
the vice president of marketing
100.0%

e. organizations objectives and
resources


Score: 2 / 2

Question 10 (2.00 points)
As one of the components of CSR, "__________ responsibility is the companys responsibility to do what is
right, just, and fair, and to avoid harm to others while pursuing its business interests.
Student response:
!ercent
Value
Correct
Response
Student
Response
Answer Choices


a. economic


b. legal
100.0%

c. ethical


d. philanthropic


e. retrocessive


Score: 2 / 2

Question 11 (2.00 points)
In its annual report to shareholders, PepsiCo publishes a _____ that states, "Our business is to increase the
value of our shareholder's investment. We do this through sales growth, cost controls and wise investment
of resources. We believe our commercial success depends upon offering quality and value to our consumers
and customers; providing products that are safe, wholesome, economically efficient and environmentally
sound; and providing a fair return to our investors while adhering to the highest standards of integrity."
Student response:
!ercent
Value
Correct
Response
Student
Response
Answer Choices


a. market opportunity analysis
(MOA)


b. research agenda
100.0%

c. mission statement


d. statement of commercial intent


e. market segmentation strategy


Score: 2 / 2

Question 12 (2.00 points)
Motel 6 advertises that its room rates are the lowest of any national chain. The company is able to keep
prices low by keeping its own costs at rock bottom. Some of Motel 6s cost-minimizing measures include
morning coffee (not breakfast), fewer in-room cable TV channels than most competitors offer, and a
bathroom that has no built-in hairdryer or Kleenex dispenser. Budget-conscious vacationers, traveling
businesspeople, and all other kinds of travelers book reservations at Motel 6 because they know exactly
what to expect: a clean room with only the most basic of amenities at a bargain price. Four Seasons Hotel &
Resorts, on the other hand, offers the highest standards of hospitality, offering elegant surroundings and the
most caring, personalized service-at one of the highest prices of any hotel chain. Motel 6 has achieved a(n)
___________ competitive advantage, and Four Seasons has achieved a(n) __________ competitive
advantage.
Student response:
!ercent
Value
Correct
Response
Student
Response
Answer Choices


a. economic; social


b. sustainable; unsustainable


c. niche; cost
100.0%

d. cost; service differentiation


e. product differentiation; cost


Score: 2 / 2

Question 13 (2.00 points)
On September 29, 2009, Starbucks introduced VIA Ready Brew in stores in its U.S. and Canadian stores.
One small packet of VIA can be mixed with 8 ounces of cold or hot water to brew a cup of coffee that is
similar in taste to a conventionally-brewed cup of Starbucks coffee. Prior to the introduction of VIA,
Starbucks coffee had never been available in instant form. By providing its loyal coffee-drinking customers
with a way to conveniently enjoy a cup of Starbucks anywhere, the company used a ______________
strategy.
Student response:
!ercent
Value
Correct
Response
Student
Response
Answer Choices
0.0%

a. market development
No . . . the target market
consisted of existing
Starbucks customers. See
pp. 22-24.


b. market penetration


c. product metamorphosis



d. product development


e. diversification


Score: 0 / 2

Question 14 (2 points)
Solitude Nurseries is a retailer that helps homeowners turn their back yards into "nature sanctuaries" where
they can meditate and ponder the wonders of nature. The company's owner believes she is able to maintain
the loyalty of the store's customers by stocking only the highest-quality merchandise and offering a
complete customer satisfaction guarantee. On which element of the marketing mix does the owner appear to
be most focused?
Student response:
!ercent
Value
Correct
Response
Student
Response
Answer Choices


a. Price


b. Distribution


c. Economies of scale


d. Promotion
100.0%

e. Product


Score: 2 / 2

Question 15 (2.00 points)
A "sales-oriented company or organization:
Student response:
!ercent
Value
Correct
Response
Student
Response
Answer Choices


a. seeks to make a profit by
creating customer value.
100.0%

b. focuses on selling what the
organization makes, not on
making what the customer
wants.


c. places importance on building
long-term relationships with
customers.


d. carefully analyzes the market
and divides it into groups of
people who are fairly similar in
terms of selected
characteristics-then it develops
marketing programs that will
bring about mutually satisfying
exchanges with one or more of
those groups.


e. recognizes that people
throughout the company (not
just the marketing department)
need to be involved in creating,
communicating, and delivering
superior customer service and
value.


Score: 2 / 2

Question 16 (2.00 points)
For every home built for a needy family by Habitat for Humanity, the Whirlpool Corporation donates a new
kitchen range and refrigerator. In case you're wondering, these are NOT "scratch and dent" appliances.
These donations are an example of how some corporations strive to _______________.
Student response:
!ercent
Value
Correct
Response
Student
Response
Answer Choices
100.0%

a. fulfill their corporate social
responsibility.


b. meet their legal obligation to
society.


c. manage their excess inventory.


d. gratify their own corporate egos.


e. carry out the Machiavellian
manifesto.


Score: 2 / 2

Question 17 (2.00 points)
Which of the following mission statements might the authors of your text say is too broad?
Student response:
!ercent
Value
Correct
Response
Student
Response
Answer Choices
0.0%

a. "A loaf of bread in every arm
(Panera)
The authors are more likely
to cite this as an example of
a mission statement that is
stated too narrowly. A
mission that is stated too
narrowly, according to the
authors, "defines the
business in terms of goods
and services rather than in
terms of the benefits
customers seek." See pp. 17-
18.


b. "To deliver exceptional vacation
experiences through the world's
best-known cruise brands that
cater to a variety of different
lifestyles and budgets, all at an
outstanding value unrivaled on
land or at sea (Carnival
Corporation)


c. "To be the leading family-
focused, value-oriented,
specialty department store
offering quality exclusive and
national brand merchandise to
the customer in environment
that is convenient, friendly, and
exciting (Kohls Illinois, Inc.)


d. "To discover, develop, and
deliver innovative medicines that
help patients prevail over serious
diseases (Bristol-Myers Squibb)



e. "To be the premier apparel
destination for young adults
(Aropostale, Inc.)


Score: 0 / 2

Question 18 (2.00 points)
The _______________ is the unique blend of product, place (distribution), promotion, and pricing strategies
designed to produce mutually satisfying exchanges between the marketer and the target market.
Student response:
!ercent
Value
Correct
Response
Student
Response
Answer Choices


a. internal environmental mix
100.0%

b. marketing mix


c. product mix


d. product line


e. market portfolio


Score: 2 / 2

Question 19 (2.00 points)
Consultants at Sandelman & Associates, a foodservice market research firm, are presenting a SWOT analysis
to McDonald's senior executives. The Sandelman consultants mention that of all QSR (quick-serve
restaurant) chains, McDonald's has THE biggest share of drive-through sales, followed (in this order) by
Wendy's, Taco Bell, Burger King, and Sonic. For management at McDonald's, their own dominance in the
drive-through segment would be classified as a(n):
Student response:
!ercent
Value
Correct
Response
Student
Response
Answer Choices
100.0%

a. strength.


b. weakness.


c. opportunity.


d. threat.


e. cost multiplier.


Score: 2 / 2

Question 20 (2 points)
A marketing plan should be written to do all of the following EXCEPT:
Student response:
!ercent
Value
Correct
Response
Student
Response
Answer Choices


a. Compare actual performance to
expected performance


b. State planned activities in a
clear and understandable way


c. Present common goals for
employees to work toward


d. Allow managers to enter the
marketplace with an awareness
of possibilities and problems
100.0%

e. Propose how the marketing
manager should go about
controlling the elements of the
external environment


Score: 2 / 2
Question 1 (2.00 points)
Claas is Europes largest manufacturer of combine harvesters (i.e., gigantic tractor-like pieces of equipment
that are used to harvest crops). As farms become larger, farming becomes more professional and more
capital intensive. As farming becomes more professional and more capital intensive, farmers are buying
more sophisticated machines, which Claas is able to supply. The large pieces of farm equipment sold by
Claas would be categorized as:
Student response:
!ercent
Value
Correct
Response
Student
Response
Answer Choices


a. component parts.


b. supplies.
0.0%

c. processed materials.
No . . . see pp. 113-115.



d. installations.


e. business services.


Score: 0 / 2

Question 2 (2.00 points)
Blogs created by consumers can be used to assess consumer attitudes toward new products as well as
customer perceptions of existing products. Marketers' ability to search for key words used in blogs is an
example of a change in the _____________ factor of the external environment.
Student response:
!ercent
Value
Correct
Response
Student
Response
Answer Choices


a. psychographic
100.0%

b. technological


c. competitive


d. demographic


e. economic


Score: 2 / 2

Question 3 (2.00 points)
With regard to a marketing plan, with which of the following statements would the authors of your text most
likely AGREE?
Student response:
!ercent
Value
Correct
Response
Student
Response
Answer Choices


a. A marketing plan needs to be
assembled based entirely upon
objective facts and figures that
have been gathered through
marketing research. Managerial
intuition and judgment should
be kept out of the process.


b. "Marketing plan is simply a
shorter, simpler way of saying
"Mission statement plus
marketing mix.
100.0%

c. A marketing plan is only as good
as the information it contains
and the effort, creativity, and
thought that went into
constructing it.


d. All of the above


e. None of the above


Score: 2 / 2

Question 4 (2.00 points)
Which of the following mission statements might the authors of your text say is too broad?
Student response:
!ercent
Value
Correct
Response
Student
Response
Answer Choices


a. "Build brand loyalty by
delighting customers with high-
quality, delicious products, `at
our place or yours, while
balancing the needs of our
employees, guests and
stockholders (Bob Evans
Farms, Inc.)


b. "To organize the worlds
information and make it
universally accessible and
useful (Google Inc.)


c. "To be a pre-eminent global
producer and merchant of iron
ore and other steel-related
products (Cliffs Natural
Resources)
100.0%

d. "To be the worlds greatest kids
brand (Toys"RUs, Inc.)


e. "To fulfill dreams through the
experience of motorcycling, by
providing to motorcyclists and to
the general public an expanded
line of motorcycles and branded
products and services in
selected market segments


Score: 2 / 2

Question 5 (2.00 points)
Cardiac Science manufactures defibrillators, heart monitoring devices, and other medical related devices.
The companys headquarters is located in New Jersey, and it operates factories in the United States, Canada,
China, and Mexico. The company sells 85 percent of its products outside the United States. A product that
Cardiac Science sells to a hospital in London is pretty much identical as a product that it sells to a hospital in
Tokyo (Japan), Pretoria (South Africa), or Sydney (Australia), and it uses the same promotional tactics in all
of its markets. Cardiac Science:
Student response:
!ercent
Value
Correct
Response
Student
Response
Answer Choices


a. cannot be described as a
"multinational corporation.
100.0%

b. uses global marketing
standardization.


c. appears to have engaged in
licensing as its primary means of
entering global markets.


d. is involved in a joint venture.


e. is following the same type of
global marketing strategy as
McDonalds does.


Score: 2 / 2

Question 6 (2.00 points)
Nestl offers Stouffer's gourmet entrees for one segment of the frozen dinner market, and Lean Cuisine for
another segment that wants both good taste and low calories. When Nestl chose to serve two well-defined
market segments and develop distinct marketing mixes for each, it was implementing a _____ strategy.
Student response:
!ercent
Value
Correct
Response
Student
Response
Answer Choices


a. market integration


b. concentrated targeting


c. undifferentiated targeting
100.0%

d. multisegment targeting


e. one-to-one marketing


Score: 2 / 2

Question 7 (2.00 points)
Holls Chocolates is a small, family-owned company whose offices and factory are located in the town of
Vienna, West Virginia. Each year, Holls sends several hundred pounds of its signature candies to specialty
candy distributors in the Canadian provinces of Nova Scotia, Quebec, and British Columbia. The Canadian
distributors then sell smaller quantities to individual candy stores and store chains. Holls method of entering
the Canadian market can best be described as ___________.
Student response:
!ercent
Value
Correct
Response
Student
Response
Answer Choices


a. countertrade
100.0%

b. exporting


c. contract manufacturing


d. licensing


e. direct investment


Score: 2 / 2

Question 8 (2.00 points)
Decisions about transportation and warehousing are part of a companys __________ strategy.
Student response:
!ercent
Value
Correct
Response
Student
Response
Answer Choices


a. promotion


b. protocol
100.0%

c. place (distribution)


d. product


e. price


Score: 2 / 2

Question 9 (2.00 points)
Strategic planning is
Student response:
!ercent
Value
Correct
Response
Student
Response
Answer Choices


a. "defining a business in terms of
goods and services rather than
in terms of the benefits
customers seek.


b. "an organizations attempt to
shape the external environment
in which it operates.


c. "the creation of a large
computerized file of customers
and potential customers profiles
and purchase patterns.


d. "the process of planning,
collecting, and analyzing data
relevant to a marketing
decision.
100.0%

e. "the managerial process of
creating and maintaining a fit
between the organizations
objectives and resources and
the evolving market
opportunities.


Score: 2 / 2

Question 10 (2.00 points)
While driving across town, you notice that your car's gas tank is almost empty. Your desired state is to feel
secure that you will not run out of gas and have to walk to the nearest station for help. Your actual state is
your knowledge that you will run out of gas if you do not put some in your tank in the next few minutes.
This imbalance between your desired state and your actual state is labeled ________________ in the
consumer decision making process.
Student response:
!ercent
Value
Correct
Response
Student
Response
Answer Choices


a. passive information gathering
100.0%

b. need recognition


c. cognitive dissonance


d. post-purchase behavior


e. None of the above are correct.


Score: 2 / 2

Question 11 (2.00 points)
The authors of your textbook discuss the BCG approach to portfolio analysis. In a marketing context, what
does "portfolio refer to?
Student response:
!ercent
Value
Correct
Response
Student
Response
Answer Choices


a. A companys short-term
investment of its excess funds in
the stock of other companies,
and perhaps in other securities
such as short-term bonds and
real estate investment trusts-in
hopes of earning a rate of return
that is greater than the interest
that a bank would pay.


b. In a large diversified company,
the shares of common stock that
it holds in itself (this is referred
to as "treasury stock in the
financial world), and the stock
that it holds in each of its
subsidiary companies. Warren
Buffett's Berkshire Hathaway
portfolio, for instance, would be
expected to include large blocks
of shares of its own stock, as
well as large blocks of shares of
its GEICO, Burlington Northern
Santa Fe, Helzberg Diamonds,
and Justin Brands subsidiaries.


c. The companys archives of its
past advertisements. GEICOs
portfolio, for instance, includes
ad copy, illustrations,
storyboards, and video
segments of all of its Gecko-
themed advertising, as well as
all of its Caveman-themed
advertising.
100.0%

d. All of the companys SBUs,
product lines, or brands.


e. None of the above.


Score: 2 / 2

Question 12 (2.00 points)
Argentina has laws that restrict "big box stores like The Home Depot and Walmart. The only way such
stores can exist in Argentina is if they do not sell either clothing or food. Also, at least 50 percent of the
merchandise sold in such stores must be manufactured in Argentina. This is an example of how which of the
following external environmental factors influences the decisions of global marketers?
Student response:
!ercent
Value
Correct
Response
Student
Response
Answer Choices


a. Economic and technological
development


b. Natural resources
100.0%

c. Political structure and actions


d. Culture


e. Demographic makeup


Score: 2 / 2

Question 13 (2.00 points)
As one of the components of CSR, "__________ responsibility is the companys responsibility to obey the
law while pursuing its business interests.
Student response:
!ercent
Value
Correct
Response
Student
Response
Answer Choices


a. economic
100.0%

b. legal


c. ethical


d. philanthropic


e. material


Score: 2 / 2

Question 14 (2.00 points)
According to the criterion of ________________, marketers need to be able to figure out roughly how many
people are in a selected market segment, as well as who those people are. Targeting a vaguely-defined
segment is likely to yield unpredictable marketing results.
Student response:
!ercent
Value
Correct
Response
Student
Response
Answer Choices
100.0%

a. identifiability and measurability


b. accountability


c. responsiveness


d. accessibility


e. causality


Score: 2 / 2

Question 15 (2.00 points)
This month, Heather Weathers, Summer Mummert, and Amber Lambert are starting a new Internet-based
business together. The online storefront, which they are naming "For Reality Lovers," will sell DVDs, posters,
figurines, collectors' cards, logo apparel, and other memorabilia from TV reality shows such as "Survivor,"
"Fear Factor," "American Idol," "The Apprentice," "The Amazing Race," "Dating in the Dark," "Project
Runway," "Big Brother," "Dancing with the Stars," "Jersey Shore, and "Tough Enough. Heather, Summer,
and Amber know that their business will be successful only under the following three conditions:


--IF reality TV continues to be popular with a large portion of the viewing public.


--IF the appropriate computer hardware and software is available to them, they are able to keep up with
ever-changing technology.


--IF they learn and abide by the many laws that apply to e-commerce, such as those that protect the privacy
of customers' information and those that pertain to collecting state sales tax.


Of the six external environmental factors that are presented in your text, which three appear to have the
greatest impact on Heather, Summer, and Ambers business? That is, which three are illustrated by the
above issues?
Student response:
!ercent
Value
Correct
Response
Student
Response
Answer Choices


a. Social, technological, and
economic


b. Political/legal, technological, and
demographic


c. Competitive, social, and
technological


d. Social, political/legal, and
demographic
100.0%

e. Social, technological, and
political/legal.


Score: 2 / 2

Question 16 (2.00 points)
Which of the following does NOT describe the business market, as compared to the consumer market?
Student response:
!ercent
Value
Correct
Response
Student
Response
Answer Choices



a. A business buyer is much less
likely to try to negotiate a price
than is a consumer.


b. Business customers tend to be
more geographically
concentrated, and customers in
consumer markets tend to be
more geographically dispersed.


c. Business markets are served by
a more direct channel of
distribution than are consumer
markets.


d. Purchasing by businesses is a
more formal process than
purchasing by consumers.
0.0%

e. Business marketers tend to have
far fewer customers than
consumer marketers.
This is a valid statement. See
pp. 110-113.


Score: 0 / 2

Question 17 (2.00 points)
__________ is how consumers perceive themselves in terms of attitudes, perceptions, beliefs, and self-
evaluations.
Student response:
!ercent
Value
Correct
Response
Student
Response
Answer Choices


a. Socialization


b. Personality


c. Lifestyle


d. Normalization
100.0%

e. Self-concept


Score: 2 / 2

Question 18 (2.00 points)
The U.S. Postal Service (USPS, also known as "the post office") claims that its Express Mail overnight
letter delivery service is comparable to what is offered by FedEx and that its prices are lower than FedExs.
Yet, on an average day, FedEx carries about ten times as many overnight letters (between U.S. locations) as
the USPS. Which of the following statements helps to explain this situation?
Student response:
!ercent
Value
Correct
Response
Student
Response
Answer Choices


a. The USPS is perceived as
offering greater customer value.
100.0%

b. FedEx is perceived as offering
greater customer value.


c. FedEx and the USPS are
perceived as offering the same
customer value.


d. Customer value is not an issue
in deciding which express-
delivery service to use.


e. The USPS should lower its prices
even further to increase market
share.


Score: 2 / 2

Question 19 (2.00 points)
Rush Express, a North American delivery service, wants to expand service into South America. If it were to
do so, such an expansion would be classified as ___________ in Ansoffs Product-Market Matrix.
Student response:
!ercent
Value
Correct
Response
Student
Response
Answer Choices


a. diversification
0.0%

b. market penetration
No . . . see pp. 22-24. Think
"new market."


c. product development



d. market development


e. market annexation


Score: 0 / 2

Question 20 (2.00 points)
Which of the following is TRUE with regard to a buying center?
Student response:
!ercent
Value
Correct
Response
Student
Response
Answer Choices
100.0%

a. A gatekeeper may be a
secretary or administrative
assistant.


b. "Deciders and "purchasers
both work in the purchasing
department; the decider is the
purchasers boss.


c. If management has not
appointed a committee to make
a purchase decision, then there
is no buying center for that
particular purchase.


d. Marketers sell to a buying
center; they are never members
of one.


e. All of the above.


Score: 2 / 2

Question 21 (2.00 points)
Through programs such as MAINTAIN (Medial Assistance for Those Who Are in Need), Global Health
Fellows, Amigos En Salud, Balance It Out, The Pfizer Education Initiative (PEI), and Science in the
Schools, Pfizer Inc. made donations to community and global health causes that exceeded $2.3 billion in
2009. These donations are an example of how some corporations strive to _______________.
Student response:
!ercent
Value
Correct
Response
Student
Response
Answer Choices
100.0%

a. fulfill their corporate social
responsibility.


b. meet their legal obligation to
society.


c. manage their excess inventory.


d. "stay on the good side of U.S.
Secretary of Health and Human
Services Kathleen Sebelius.


e. distract their critics.


Score: 2 / 2

Question 22 (2.00 points)
If a company truly accepts and embraces the marketing concept, then
Student response:
!ercent
Value
Correct
Response
Student
Response
Answer Choices


a. its management will have a ield
of Dreams orientation, whereby
they believe "If we build it, they
will come.


b. responsibility for implementing
and "living up to the marketing
concept rests entirely on the
shoulders of the firms
marketing managers.


c. its management believes that
low demand for the companys
products must mean that
salespeople must not be
approaching potential customers
aggressively enough.
100.0%

d. the company will be actively
engaged in collecting
information about customers,
competitors, and markets.


e. the companys CEO must have
been a VP of marketing before
being promoted to CEO.


Score: 2 / 2

Question 23 (2.00 points)
The U.S. government is
Student response:
!ercent
Value
Correct
Response
Student
Response
Answer Choices


a. primarily a reseller.


b. the producer of over 50% of the
nations GDP.


c. categorized as an "institution
when discussing the categories
of business customers.


d. the nations largest customer,
and the worlds fourth-largest
customer.
100.0%

e. None of the above.


Score: 2 / 2

Question 24 (2.00 points)
Hubert Neubert owns Ballistic Bazaar (a gun shop and shooting range). Hubert decides he can segment his
market into three distinct groups: those who want a weapon for home protection, those who want a weapon
for seasonal hunting of small game (a.k.a. `varmints), and collectors who enjoy owning a wide variety of
firearms for their intrinsic and aesthetic value. Thus, he segments his market into those who are seeking
safety for their household and/or family, those who are seeking recreation, and those who are seeking the
satisfaction of ownership in and of itself. Which label most closely resembles the dimension Mr. Neubert is
using to segment his market?
Student response:
!ercent
Value
Correct
Response
Student
Response
Answer Choices


a. Geographic segmentation
100.0%

b. Benefit segmentation


c. Income segmentation


d. Family life-cycle (FLC)
segmentation


e. Age segmentation


Score: 2 / 2

Question 25 (2.00 points)
Wal-Mart, Inc.'s Wal-Mart Stores division consists of its regular Walmart stores and its Walmart
Supercenters. Wal-Mart is able to live up to its claims of "Always low prices . . . and "Saving people money
so they can live better by keeping its operating costs at the lowest level in the retail industry. Through
efficient production of its own store-brand products, efficient distribution of all products to its stores, and
insistence on paying bottom dollar to suppliers of the brands it sells, Wal-Mart is able to charge consumers
the lowest possible prices at its Walmart stores and Walmart Supercenters. Wal-Mart has achieved
Student response:
!ercent
Value
Correct
Response
Student
Response
Answer Choices


a. one-to-one marketing.


b. a niche competitive advantage.


c. a service differentiation
competitive advantage.
100.0%

d. a cost competitive advantage.


e. derived demand.


Score: 2 / 2

Question 26 (2.00 points)
Product positioning is the:
Student response:
!ercent
Value
Correct
Response
Student
Response
Answer Choices


a. process of choosing the right
retail locations through which to
sell a product.


b. process of choosing a channel of
distribution through which to sell
a product.
100.0%

c. process of creating the desired
image of a firms product in
comparison with its competition.


d. process of securing the desired
shelf space for a product.


e. sales rank of a brand (such as
#1 best seller).


Score: 2 / 2

Question 27 (2.00 points)
When Virgil Sturgills wife sends him to the store to buy paper towels, Coke, and cereal, he makes a
"surgical strike. Maintaining a stride that matches the tempo of the music on his iPod, he heads for the
paper products aisle first. There, he reaches for the largest package of Bounty that is available, and puts it
in the cart in one deft movement-without any discernible slowing or stopping on the part of his feet or the
shopping cart. The scene repeats itself in the breakfast foods aisle-a box of Honey Nut Cheerios makes its
way from shelf to moving cart in a clean, arc-like motion. In the carbonated beverage aisle, Virgil has to
stop the cart momentarily, but only because he needs both hands to hoist the 24-pack of Coca-Cola Classic
into the cart. This scenario illustrates:
Student response:
!ercent
Value
Correct
Response
Student
Response
Answer Choices


a. extensive decision making.


b. decelerated decision making.


c. limited decision making.
100.0%

d. routinized response behavior.


e. tertiary decision making.


Score: 2 / 2

Question 28 (2.00 points)
The current "official American Marketing Association (AMA) definition of marketing, as quoted by the
authors of your text, is: "Marketing is the activity, set of institutions, and processes for creating,
communicating, delivering, and __________ customers, clients, partners, and society at large.
Student response:
!ercent
Value
Correct
Response
Student
Response
Answer Choices


a. selling tangible goods to
100.0%

b. exchanging offerings that have
value for


c. exaggerating the properties and
capabilities of products and
services for


d. financing the purchase of
products by


e. maintaining the illusion of
satisfaction for


Score: 2 / 2

Question 29 (2.00 points)
Consultants at Insula Research are presenting a SWOT analysis of McDonald's to McDonald's senior
executives. The Insula consultants mention that of all QSR (quick-service restaurant) chains, Wendys has
the fastest average service time in the drive-through lane, averaging 134.09 seconds per vehicle. Insulas
research study found Wendys closest competitors in average service time to be Del Taco (at 150.3 seconds
per vehicle), Taco Bell (at 163.2 seconds per vehicle), Burger King (at 166.7 seconds per vehicle), and
Chick-fil-A (at 167.6) seconds per vehicle. From the perspective of top management at McDonalds, the fact
that their own average drive-through service time does not rank among the five best in the industry would
be classified as a(n):
Student response:
!ercent
Value
Correct
Response
Student
Response
Answer Choices


a. strength.
100.0%

b. weakness.


c. opportunity.


d. threat.


e. value proposition.


Score: 2 / 2

Question 30 (2.00 points)
Skye Frye is a new-vehicle salesperson at Jim Finney GMC-Mazda. Skye is very concerned about treating
each customer fairly and developing long-term relationships with them. She will not try to sell a customer a
vehicle that the customer clearly cannot afford, and she contacts each customer personally within two weeks
after a purchase to say "Thank you and to make sure the vehicle is meeting the customers expectations. If
the customer mentions a problem with the vehicle during Skyes follow-up call, Skye will contact Jim
Finneys service department to make an appointment for the customer, rather than simply suggesting that
the customer call the service department. On more than one occasion, Skye has had her husband drop her
at an elderly customers home so that she could drive the customers vehicle to the dealerships service
department herself. She told your professor: "People can bypass me and buy a car on the Internet or
through an auto broker. As a salesperson, I only deserve my commission if Im genuinely helpful and if I
treat each customer with dignity and respect. Skye Frye is most likely at the ______________ stage of
ethical development.
Student response:
!ercent
Value
Correct
Response
Student
Response
Answer Choices


a. preconventional morality


b. quixotic morality


c. conventional morality
100.0%

d. postconventional morality


e. compromised morality


Score: 2 / 2
Question 1 (2.00 points)
Rhett Lett used to sell office furniture and supplies, but now he is semi-retired and started a cleaning
service. He specializes in cleaning fire- and water-damaged structures. Rhett is finding that selling services
differs greatly from selling office furniture and supplies because cleaning services are:
Student response:
!ercent
Value
Student
Response
Answer Choices


a. homogeneous.


b. synergistic.
100.0%

c. intangible.


d. in the introductory stage of the product life
cycle.


e. convenience products.


Score: 2 / 2

Question 2 (2.00 points)
All other things being equal, when brand awareness is high, and the brand is perceived to be of high quality,
and customers of the brand are extremely loyal to it, the brand can be said to have __________.
Student response:
!ercent
Value
Student
Response
Answer Choices
100.0%

a. high brand equity


b. a monopoly


c. brand security


d. synonymity


e. economies of scale


Score: 2 / 2

Question 3 (2.00 points)
Marketing researchers use the Internet to:
Student response:
!ercent
Value
Student
Response
Answer Choices


a. facilitate collaboration between the client
and the research supplier in the
management of a research project.


b. conduct online focus groups.


c. monitor consumer-generated media (CGM),
such as blogs and YouTube videos.


d. put employees in direct contact with
customers to gather input regarding
products and services.
100.0%

e. do all of the above.


Score: 2 / 2

Question 4 (2.00 points)
Which of the following mission statements might the authors of your text say is too broad?
Student response:
!ercent
Value
Student
Response
Answer Choices


a. "To equip people to make a difference in
their world-we do this by creating
outstanding products and by trying to make
a difference in the communities in which we
live and work (Timberland).
0.0%

b. "To be a trusted, respected, and ethical food
industry leader that excels at bringing
delicious and nutritious meat and specialty
products to millions every day while setting
industry standards for corporate social
responsibility (Smithfield Foods)
The authors are more likely to cite this
as an example of a mission statement
that is 8tated well. It identifies the
business that the company perceives
itself to be in, and it reflects that the
company understands what benefits
customers seek. See pp. 17-18.


c. "To facilitate the security of our members,
associates, and their families through
provision of a full range of highly
competitive financial products and services;
in so doing, we seek to be the provider of
choice for the military community. (USAA)


d. "To provide personal vehicle owners and
enthusiasts with the vehicle related products
and knowledge that fulfill their wants and
needs at the right price. Our friendly,
knowledgeable and professional staff will
help educate, inspire and problem-solve for
our customers. (Advance Auto Parts)


e. "To offer lending and investment products
that serve low- and moderate-income
individuals and families; to improve
underserved low- and moderate-income
communities; to create sustainable practices
in the long haul (Bank of America)


Score: 0 / 2

Question 5 (2.00 points)
Companies that are MOST likely to succeed in the introduction of new products typically have:
Student response:
!ercent
Value
Student
Response
Answer Choices


a. a project-based team approach to new-
product development.


b. a vision of what the market will be like in the
future.


c. a history of carefully listening to customers.
0.0%

d. a long-term commitment to innovation and
new-product development.
True . . . but there's more. See pp. 172-
179.


e. all of the above.


Score: 0 / 2

Question 6 (2.00 points)
___________________ is the development of a specific marketing mix to influence potential customers
overall perception of a brand, product line, or organization in general. It is related to "the place that a
product occupies in consumers minds relative to competitive offerings.
Student response:
!ercent
Value
Student
Response
Answer Choices


a. Multisegment targeting


b. Segment profiling and analysis


c. Undifferentiated targeting


d. Optimization
100.0%

e. Positioning


Score: 2 / 2

Question 7 (2.00 points)
_____________ specifies the research questions to be answered, how and when the data will be gathered,
and how the data will be analyzed.
Student response:
!ercent
Value
Student
Response
Answer Choices


a. A secondary agenda


b. The sales manager


c. A sampling frame
100.0%

d. The research design


e. The mission statement


Score: 2 / 2

Question 8 (2.00 points)
Using the BCG (Boston Consulting Group) matrix, how would Procter & Gamble categorize itself?
Student response:
!ercent
Value
Student
Response
Answer Choices


a. As a dog


b. As a star
0.0%

c. As a cash cow
A large, diversified corporation such as
!&G WOULD NOT categorize its overall
organization using the BCG matrix; it
would categorize each of its SBUs (e.g.,
global beauty care, global health & well
being, global household care), or its
product lines (e.g., skin care,
antiperspirants, oral care), or its
individual brands (e.g., Crest, !ampers,
Downy, Old Spice, Gillette). See pp. 23-
24.


d. As a problem child


e. As none of the above.


Score: 0 / 2

Question 9 (2.00 points)
Services sometimes have important characteristics that customers cannot evaluate-even after purchase-
because they do not have the knowledge or experience to judge those characteristics accurately. The
authors of your textbook refer to such characteristics as
Student response:
!ercent
Value
Student
Response
Answer Choices


a. search qualities


b. tandem qualities


c. experience qualities
100.0%

d. credence qualities


e. salient qualities


Score: 2 / 2

Question 10 (2.00 points)
Krogers "Big K cola, Targets "Archer Farms snacks, and Walmarts "Ol Roy dog food are examples of
__________ brands.
Student response:
!ercent
Value
Student
Response
Answer Choices
0.0%

a. manufacturers
No . . . each is the brand of one specific
retailer. "!epsi," "!lanters," and
"!urina One" are examples of
manufacturers' brands. See pp. 162-
166.


b. national


c. generic


d. private


e. gray market


Score: 0 / 2

Question 11 (2.00 points)
Hank Swank is a calculating, self-centered salesperson. He never engages in relationship marketing. He
looks on any sales situation as a "win-lose" situation, and he believes any method he can use to close the
deal is justified by the outcome. Hank is most likely at the __________ stage of ethical development.
Student response:
!ercent
Value
Student
Response
Answer Choices


a. preconventional morality


b. transcendant universal


c. conventional morality


d. postmodern enlightened
0.0%

e. postconventional morality
Most definitely not. See p. 33.


Score: 0 / 2

Question 12 (2.00 points)
A marketing executive who believes in the "total CSR approach to social responsibility will adopt the
perspective of the "pyramid of corporate social responsibility, which contains all of the following
components EXCEPT:
Student response:
!ercent
Value
Student
Response
Answer Choices


a. economic responsibility.


b. legal responsibility.
100.0%

c. cultural responsibility.


d. ethical responsibility.


e. philanthropic responsibility.


Score: 2 / 2

Question 13 (2.00 points)
With regard to marketing by nonprofit organizations, with which of the following statements would the
authors of your text likely AGREE?
Student response:
!ercent
Value
Student
Response
Answer Choices


a. The target market of a nonprofit
organization may consist of people who do
not care about-or are even opposed to-
receiving the services that the organization
offers.


b. Nonprofit organizations often market
products or services that are by definition
"very low involvement.


c. The benefits of nonprofit organizations
product offerings may be more difficult to
communicate than those of for-profit
marketers.


d. All of the above
0.0%

e. None of the above
At least one of "a" through "d" is valid.
See pp. 196-199.


Score: 0 / 2

Question 14 (2.00 points)
Big Regional Bank, Inc. has over 200 branches spread throughout a six-state area. Katie Beatty is vice
president of marketing at Big Regional.


Katie keeps track of the marketing actions of competing banks by reading each issue of the ABA (American
Banking Association) ank Marketing magazine. She also scours each issue of Advertising Age, Adweek,
Communication World, to , Media Week, and Marketing ews for news of competitors new and upcoming
campaigns. In addition to seeing what her rivals are doing, shes able to maintain a flow of fresh ideas by
reading these magazines. She receives a "heads up regarding impending marketing-related legislation by
reading these magazines, as well.


Katie also peruses each weekday issue of The Wall Street Journal, each weekday issue of nvestors usiness
Daily, each bi-weekly issue of The Economist, and each semi-monthly issue of orbes in order to stay
informed of developments in the regional, national, and global economy. She supplements her reading by
regularly watching "The Kudlow Report and "Street Signs on CNBC and "The Cavuto Report and "The
Willis Report on Fox Business Channel, and she tunes into "The Wall Street Journal Report on a local radio
station while shes driving to work. From these publications, she is able to amass even more knowledge of
industry-related events in the political/legal arena as well as staying up to date on whats happening in the
economy.


Since sixty percent of her banks customers are small businesses, Katie also reads each monthly issue of the
magazines nc., ortune Small usiness, Entrepreneur, and lack Enterprise so that she can be aware of
matters that may be of great importance to customers and potential customers. Local publications such as
West Virginia Executive and Pittsburgh magazine keep her "in the know pertaining to developments in the
business environments of her banks specific geographic markets.


To stay on top of technological developments, Katie reads ast Company, usiness 2.0, and CRM to stay on
top of major technological developments. Her regular reading of ortune magazine, Harvard usiness
Review, and loomberg usinessWeek also help Katie maintain up-to-date knowledge of a variety of
business-related matters.


And, to remain knowledgeable of social issues and social attitudes that can potentially affect her employers
business, Katie reads ew Yorker, Vanity air, illboard, Rolling Stone, ewsweek, Time, and The Atlantic.


As described above, Katies viewing, reading, and listening activities are:
Student response:
!ercent
Value
Student
Response
Answer Choices


a. redundant in her role as VP of marketing;
thats the CEOs job.


b. a symptom of obsessive compulsive
disorder.
100.0%

c. an example of environmental scanning.


d. an example of primary research.


e. an example of observational research.


Score: 2 / 2

Question 15 (2.00 points)
Argentina has laws that restrict "big box stores like The Home Depot and Walmart. The only way such
stores can exist in Argentina is if they do not sell either clothing or food. Also, at least 50 percent of the
merchandise sold in such stores must be manufactured in Argentina. This is an example of how which of the
following external environmental factors influences the decisions of global marketers?
Student response:
!ercent
Value
Student
Response
Answer Choices


a. Economic and technological development


b. Natural resources
100.0%

c. Political structure and actions


d. Culture


e. Demographic makeup


Score: 2 / 2

Question 16 (2.00 points)
When Krispy Kreme decided to expand its operation internationally, it chose Canada as its first new market.
In accordance with the policy of minimizing investment costs and risk, the company sold the right to
manufacture and sell its doughnuts to Canadians. In other words, Krispy Kreme used:
Student response:
!ercent
Value
Student
Response
Answer Choices


a. contract manufacturing.


b. direct investment.


c. exporting.


d. a joint venture.
100.0%

e. licensing.


Score: 2 / 2

Question 17 (2.00 points)
While driving across town, you notice that your car's gas tank is almost empty. Your desired state is to feel
secure that you will not run out of gas and have to walk to the nearest station for help. Your actual state is
your knowledge that you will run out of gas if you do not put some in your tank in the next few minutes.
This imbalance between your desired state and your actual state is labeled ________________ in the
consumer decision making process.
Student response:
!ercent
Value
Student
Response
Answer Choices


a. passive information gathering
100.0%

b. need recognition


c. cognitive dissonance


d. post-purchase behavior


e. None of the above are correct.


Score: 2 / 2

Question 18 (2.00 points)
Opinion leaders are:
Student response:
!ercent
Value
Student
Response
Answer Choices


a. wealthy, well-educated individuals.


b. experts on all high-involvement consumer
goods.


c. usually the same individuals for all social
classes.
100.0%

d. people who influence others.


e. easy to locate and target.


Score: 2 / 2

Question 19 (2.00 points)
The Triple S Health Spa has outgrown its building. Its owner wants to know how its customers will react to a
move and if the addition of new services will make the new location more appealing to its loyal patrons. The
owner has surveyed several customers to gauge their reaction to various potential locations as well as their
reactions to various prospective service mixes. The survey results are ______________ data.
Student response:
!ercent
Value
Student
Response
Answer Choices


a. observation


b. secondary


c. collaborative
100.0%

d. primary


e. posthumous


Score: 2 / 2

Question 20 (2.00 points)
Marketing managers can use marketing research to:
Student response:
!ercent
Value
Student
Response
Answer Choices


a. improve the quality of their decision making.


b. find out why a marketing plan failed.


c. learn how to more efficiently retain
customers.


d. better understand the ever-changing
marketplace.
100.0%

e. do all of the above.


Score: 2 / 2

Question 21 (2.00 points)
Two of the most important pieces of equipment at Cobalt Coal Corporations Westchester mine at Big Sandy,
West Virginia are its Joy continuous miner (for which it paid over $500,000) and its Caterpillar diesel-
powered generator (for which it paid another $500,000). The company would not be able to extract coal
from the McDowell County mountainside without the cuts made by the 40-foot continuous miner, and the
"miner would be useless without the electricity that is provided by the generator. CEO Mike Crowder hopes
that both pieces of equipment will operate reliably for years to come, and on its financial statements Cobalt
"depreciates them rather than "expensing them. As business products, the continuous miner would be
categorized as __________; the generator would be categorized as __________.
Student response:
!ercent
Value
Student
Response
Answer Choices


a. accessory equipment; an installation
100.0%

b. an installation; also an installation


c. an installation; an accessory part


d. a component part; an MRO item


e. accessory equipment; also accessory
equipment


Score: 2 / 2

Question 22 (2.00 points)
The authors of your text use the "gap model to illustrate discrepancies that can cause problems in service
delivery. These same discrepancies can influence customer evaluations of service quality in a negative way.
Which of the following is NOT one of the gaps in the gap model?
Student response:
!ercent
Value
Student
Response
Answer Choices


a. The gap between what the company
provides and what the customer is told it
provides


b. The gap between what management thinks
customers want and the quality
specifications management develops


c. The gap between the service quality
specifications and the service that is actually
provided


d. The gap between the quality of services
customers want and the quality of service
they receive
100.0%

e. The gap between the level of service that is
provided by a union employee and the level
of service that is provided by a nonunion
employee


Score: 2 / 2

Question 23 (2.00 points)
_______________ is the process of grouping consumers into market segments based on what they seek
from the product.
Student response:
!ercent
Value
Student
Response
Answer Choices


a. Geographic segmentation


b. Psychographic segmentation


c. Demographic segmentation


d. Usage-rate segmentation
100.0%

e. Benefit segmentation


Score: 2 / 2

Question 24 (2.00 points)
A ___________ is the part of the brand that cannot be spoken.
Student response:
!ercent
Value
Student
Response
Answer Choices
100.0%

a. brand mark


b. service mark


c. trademark


d. brand name.


e. certification mark


Score: 2 / 2

Question 25 (2.00 points)
Sam and Phil Barshop developed the La Quinta Inn chain of hotels with the aim of satisfying the lodging
needs of a specific type of traveler: the cost-conscious white-collar business traveler. By focusing upon this
market segment, the Barshop brothers were practicing _____________ targeting.
Student response:
!ercent
Value
Student
Response
Answer Choices


a. undifferentiated


b. multisegment
100.0%

c. concentrated


d. aggregated


e. one-to-one


Score: 2 / 2

Question 26 (2.00 points)
Jan Karon's best-selling novels about Mitford (a fictional town in the mountains of North Carolina) caught the
attention of the Hallmark Company. Hallmark developed a new line of products for its Hallmark Gold Crown
Stores nationwide. The new product line includes hundreds of Mitford-inspired items that authentically bring
"the little town with the big heart" into tangible reality. The products include greeting cards, partyware, gift
wrap, mugs, ornaments, and puzzles for Hallmark's existing customers. Hallmark used a _____________
strategy.
Student response:
!ercent
Value
Student
Response
Answer Choices


a. market development


b. market penetration


c. product penetration
100.0%

d. product development


e. diversification


Score: 2 / 2

Question 27 (2.00 points)
_____ is the function that links the consumer, customer, and public to the marketer through information.
Student response:
!ercent
Value
Student
Response
Answer Choices


a. Artificial intelligence


b. Decision framing


c. A feasibility study
100.0%

d. Marketing research


e. Decision support


Score: 2 / 2

Question 28 (2.00 points)
During the growth stage of the product life cycle:
Student response:
!ercent
Value
Student
Response
Answer Choices


a. manufacturers are relatively unconcerned
with distribution strategy.


b. demand for the product is limited to high-
income consumers.


c. competition decreases.
100.0%

d. industry profits peak.


e. the late majority begin to adopt the product.


Score: 2 / 2

Question 29 (2.00 points)
Making sure products and services are available when and where customers want them is the job of which
element of the marketing mix?
Student response:
!ercent
Value
Student
Response
Answer Choices


a. Advertising strategies


b. Production strategies


c. Product strategies


d. Promotion strategies
100.0%

e. Distribution strategies


Score: 2 / 2

Question 30 (2.00 points)
Buyers in the business market tend to __________ than buyers who are consumers.
Student response:
!ercent
Value
Student
Response
Answer Choices


a. use reciprocity less often


b. purchase in smaller quantities
0.0%

c. generate more stable demand trends
No . . . see pp. 110-113.


d. be more geographically concentrated


e. be less aware of the meaning of NAICS
codes


Score: 0 / 2

Question 31 (2.00 points)
The manufacturer of Omega watches has a market orientation and strives to live by the philosophy of the
marketing concept. Which of the following describes the first action the company would most likely take if it
learned its customers were dissatisfied with its watches?
Student response:
!ercent
Value
Student
Response
Answer Choices


a. Hire more salespeople


b. Find ways to decrease its overhead


c. Increase the frequency and intensity of its
advertising


d. Try to increase the number of jewelry stores
that carry Omega watches
100.0%

e. Conduct research to try to determine
whether customer needs or wants have
changed


Score: 2 / 2

Question 32 (2.00 points)
As a marketing research method, telephone interviews (where researchers employ interviewers to call
consumers and ask them a planned set of questions):
Student response:
!ercent
Value
Student
Response
Answer Choices


a. are prohibited by current federal "Do Not
Call law.
100.0%

b. are one of the speediest ways to collect
data.


c. usually do not involve nationwide samples.


d. have a response rate close to 100 percent.


e. are THE highest-cost method of collecting
data.


Score: 2 / 2

Question 33 (2.00 points)
One of the most expensive refrigerators in the world is the Sub-Zero brand. Many consumers believe it to be
the best brand made. For people who insist on having the best-equipped kitchen that money can buy and
will accept nothing less, the Sub-Zero refrigerator is an example of a(n) __________ product.
Student response:
!ercent
Value
Student
Response
Answer Choices


a. heterogeneous shopping
100.0%

b. specialty


c. homogeneous shopping


d. convenience


e. generic


Score: 2 / 2

Question 34 (2.00 points)
Motel 6 advertises that its room rates are the lowest of any national chain. The company is able to keep
prices low by keeping its own costs at rock bottom. Some of Motel 6s cost-minimizing measures include
morning coffee (not breakfast), fewer in-room cable TV channels than most competitors offer, and a
bathroom that has no built-in hairdryer or Kleenex dispenser. Budget-conscious vacationers, traveling
businesspeople, and all other kinds of travelers book reservations at Motel 6 because they know exactly
what to expect: a clean room with only the most basic of amenities at a bargain price. Four Seasons Hotel &
Resorts, on the other hand, offers the highest standards of hospitality, offering elegant surroundings and the
most caring, personalized service-at one of the highest prices of any hotel chain. Motel 6 has achieved a(n)
___________ competitive advantage, and Four Seasons has achieved a(n) __________ competitive
advantage.
Student response:
!ercent
Value
Student
Response
Answer Choices


a. economic; social


b. sustainable; unsustainable


c. niche; cost
100.0%

d. cost; service differentiation


e. product differentiation; cost


Score: 2 / 2

Question 35 (2.00 points)
Carly Farley bought a bottle of Heinz "Hot & Spicy ketchup with TABASCO sauce. Since both brands
were prominently displayed on the label, this would be an example of:
Student response:
!ercent
Value
Student
Response
Answer Choices


a. private-label branding.


b. complementary branding.


c. product line contraction.


d. piggyback branding.
100.0%

e. cobranding.


Score: 2 / 2

Question 36 (2.00 points)
A written document that acts as a guidebook of marketing activities for the marketing manager is known as
the _________.
Student response:
!ercent
Value
Student
Response
Answer Choices


a. strategy manifesto


b. corporate charter


c. vision statement


d. mission statement
100.0%

e. marketing plan


Score: 2 / 2

Question 37 (2.00 points)
Which of the following statements about how adopters participate in the diffusion process is FALSE?
Student response:
!ercent
Value
Student
Response
Answer Choices


a. Early adopters may also be opinion leaders.
100.0%

b. The late majority are the last consumers to
adopt a new product.


c. The early majority tend to adopt a consumer
product during the growth stage of the
product life cycle.


d. Skepticism is a dominant characteristic of
members of the late majority.


e. Innovators are the first to purchase a new
product.


Score: 2 / 2

Question 38 (2.00 points)
Which of the following activities is most likely to be an example of routine response behavior?
Student response:
!ercent
Value
Student
Response
Answer Choices


a. The purchase of a three-week vacation
cruise


b. The purchase of a new propane grill for
$6,000


c. The purchase of your first copy machine for
your office at home


d. The purchase of an infant car seat for your
first child
100.0%

e. The purchase of toilet paper


Score: 2 / 2

Question 39 (2.00 points)
As one of the components of CSR, "__________ responsibility is the companys responsibility to make
profits for its owners.
Student response:
!ercent
Value
Student
Response
Answer Choices
100.0%

a. economic


b. legal


c. ethical


d. philanthropic


e. jurisdictional


Score: 2 / 2

Question 40 (2.00 points)
Which of the following is TRUE with regard to a buying center?
Student response:
!ercent
Value
Student
Response
Answer Choices
100.0%

a. A gatekeeper may be a secretary or
administrative assistant.


b. "Deciders and "purchasers both work in
the purchasing department; the decider is
the purchasers boss.


c. If management has not appointed a
committee to make a purchase decision,
then there is no buying center for that
particular purchase.


d. Marketers sell to a buying center; they are
never members of one.


e. All of the above.


Score: 2 / 2

Question 1 (2.00 points)
A _____________ is an off-price retailer that is owned and operated by a manufacturer and carries one line
of merchandise-its own.
Student response:
!ercent
Value
Correct
Response
Student
Response
Answer Choices


a. mass merchandiser
100.0%

b. factory outlet


c. wholesale club


d. discount store


e. bargain basement store


Score: 2 / 2

Question 2 (2.00 points)
Which of the following mission statements might the authors of your text say is too broad?
Student response:
!ercent
Value
Correct
Response
Student
Response
Answer Choices


a. "To equip people to make a
difference in their world-we do
this by creating outstanding
products and by trying to make
a difference in the communities
in which we live and work
(Timberland).
0.0%

b. "To solve unsolved problems
innovatively (3M)
True . . . but in this list of
options, this is not the only
mission statement that is too
broad by the authors'
standards. See pp. 17-18.


c. "To enable economic growth
through infrastructure and
energy development, and to
provide solutions that protect
people and preserve the planet
(Caterpillar)



d. All of the above


e. None of the above


Score: 0 / 2

Question 3 (2.00 points)
Peachtree Windows has no windows in inventory waiting for someone to order them. It does not make a
window until it is ordered. It is able to make 27,000 different window configurations and promises delivery in
five working days because of the excellent relationships it has established with its suppliers. This partnering
relationship that Peachtree Windows has established with its suppliers and contractors shows that Peachtree
uses:
Student response:
!ercent
Value
Correct
Response
Student
Response
Answer Choices
100.0%

a. supply chain management.


b. yield management


c. channel intimidation.


d. voodoo manufacturing.


e. "push distribution.


Score: 2 / 2

Question 4 (2.00 points)
The Rashid brothers own a hotel on the southern edge of the Charleston metro area. Their hotel is popular
among business travelers and, in particular, among business travelers who are visiting southwestern West
Virginia on coal-related business. Since 2009, the federal government has delayed the issuance and renewal
of surface coal mining permits. Now, because of a reduction in the number of active mines in southwestern
West Virginia, the coal industry is drawing fewer out-of-town and out-of-state business travelers-and the
Rashid brothers hotel is no longer at or near full occupancy on most weeknights. If the Rashid brothers were
to conduct a SWOT analysis, they would most likely identify the federal government as a:
Student response:
!ercent
Value
Correct
Response
Student
Response
Answer Choices


a. strength.


b. weakness.


c. opportunity.
100.0%

d. threat.


e. competitive factor.


Score: 2 / 2

Question 5 (2.00 points)
Services often have important characteristics that customers can evaluate only after they use the service.
Such a characteristic of a service-such as the level of professionalism of a limousine service-is called a
Student response:
!ercent
Value
Correct
Response
Student
Response
Answer Choices


a. search quality


b. tandem quality
100.0%

c. experience quality


d. credence quality


e. salient quality


Score: 2 / 2

Question 6 (2.00 points)
Motel 6 advertises that its room rates are the lowest of any national chain. The company is able to keep
prices low by keeping its own costs at rock bottom. Some of Motel 6s cost-minimizing measures include
morning coffee (not breakfast), fewer in-room cable TV channels than most competitors offer, and a
bathroom that has no built-in hairdryer or Kleenex dispenser. Budget-conscious vacationers, traveling
businesspeople, and all other kinds of travelers book reservations at Motel 6 because they know exactly
what to expect: a clean room with only the most basic of amenities at a bargain price. Four Seasons Hotel &
Resorts, on the other hand, offers the highest standards of hospitality, offering elegant surroundings and the
most caring, personalized service-at one of the highest prices of any hotel chain. Motel 6 has achieved a(n)
___________ competitive advantage, and Four Seasons has achieved a(n) __________ competitive
advantage.
Student response:
!ercent
Value
Correct
Response
Student
Response
Answer Choices


a. economic; social


b. sustainable; unsustainable


c. niche; cost
100.0%

d. cost; service differentiation


e. product differentiation; cost


Score: 2 / 2

Question 7 (2.00 points)
Rush Express, a North American delivery service, wants to expand service into South America. If it were to
do so, such an expansion would be classified as ___________ in Ansoffs Product-Market Matrix.
Student response:
!ercent
Value
Correct
Response
Student
Response
Answer Choices


a. diversification


b. market penetration


c. product development
100.0%

d. market development


e. market annexation


Score: 2 / 2

Question 8 (2.00 points)
Regarding the transportation component of supply chain logistics, when we refer to "capability, we are
talking about:
Student response:
!ercent
Value
Correct
Response
Student
Response
Answer Choices
100.0%

a. a carrier's ability to provide the
appropriate equipment and
conditions for moving specific
kinds of goods.


b. the consistency with which a
carrier delivers goods on time.


c. a carrier's ability to move goods
over a specific route.


d. the relative ease with which a
shipment can be located and
transferred.


e. the total amount a specific
carrier charges to move the
product.


Score: 2 / 2

Question 9 (2.00 points)
Making sure products and services are available when and where customers want them is the job of which
element of the marketing mix?
Student response:
!ercent
Value
Correct
Response
Student
Response
Answer Choices


a. Advertising strategies


b. Production strategies


c. Product strategies


d. Promotion strategies
100.0%

e. Distribution strategies


Score: 2 / 2

Question 10 (2.00 points)
Ethical development can be thought as having three levels. The most "evolved level is the _____ stage.
Student response:
!ercent
Value
Correct
Response
Student
Response
Answer Choices


a. preconventional morality


b. transconventional morality


c. conventional morality
100.0%

d. postconventional morality


e. panconventional morality


Score: 2 / 2

Question 11 (2.00 points)
Which of the following is NOT true with regard to consumer involvement in the purchase process?
Student response:
!ercent
Value
Correct
Response
Student
Response
Answer Choices


a. If a purchase is a high-
involvement purchase, the
consumer will probably engage
in extensive information
search-both internal and
external-before deciding which
brand to buy.
100.0%

b. Marketers can never raise
consumers involvement level in
the purchase of low-involvement
products.


c. The circumstances of a purchase
may temporarily transform a
low-involvement decision into a
high-involvement decision.


d. Some product and service
categories-such as homes,
automobiles, fashion, and choice
of a college or university to
attend-tend to be high-
involvement decisions for most
consumers, but there usually
are a few purchasers who treat
the decision as a low-
involvement decision.


e. If a consumer experiences
cognitive dissonance after
purchasing a product, the
purchase decision was probably
a high-involvement one.


Score: 2 / 2

Question 12 (2.00 points)
Chilis Bar & Grill--a chain of over 1,400 restaurants--is owned by Brinker International. However, the Chilis
on Richmond Road in Lexington, Kentucky is not owned by Brinker. It is owned and operated by former
Milwaukee Bucks forward/shooting guard Ulysses "Junior Bridgeman. Mr. Bridgeman (or, to be technically
precise, Mr. Bridgemans company, ERJ Inc., which is named after his daughter Eden and his sons Ryan and
Justin) has paid Brinker International an agreed-upon initial fee of a few hundred thousand dollars, as well
as paying royalties on the restaurants gross sales on a continual basis. In return, Mr. Bridgeman receives
the right to use Chilis business format, and he agrees to follow a thick book of rules as to how the location
is to be operated. In this situation, Brinker International is the __________ and ERJ, Inc. is the
__________.
Student response:
!ercent
Value
Correct
Response
Student
Response
Answer Choices


a. producer/manufacturer; supply
chain
100.0%

b. franchisor; franchisee


c. retailer; wholesaler


d. parent corporation; subsidiary


e. franchisee; franchisor


Score: 2 / 2

Question 13 (2.00 points)
This month, Sara Herrera is starting a new Internet-based business. The online storefront-which she is
naming "For Reality Lovers"-will sell DVDs, posters, figurines, collectors' cards, logo apparel, and other
memorabilia from TV reality shows such as "Survivor," "Fear Factor," "American Idol," "The Apprentice,"
"The Amazing Race," "Dating in the Dark," "Project Runway," "Big Brother," "Dancing with the Stars,"
"Jersey Shore, and "Tough Enough. Sara knows that her business will be successful only under the
following three conditions:


--IF reality TV continues to be popular with a large portion of the viewing public.


--IF the appropriate computer hardware and software is available to her, and she is able to keep up with
ever-changing technology.


--IF she learns and abides by the many laws that apply to e-commerce, such as those that protect the
privacy of customers' information and those that pertain to collecting state sales tax.


Of the six external environmental factors that are presented in your text, which three appear to have the
greatest impact on Saras business? That is, which three are illustrated by the above issues?
Student response:
!ercent
Value
Correct
Response
Student
Response
Answer Choices


a. Social, technological, and
economic


b. Political/legal, technological, and
demographic


c. Competitive, social, and
technological


d. Social, political/legal, and
demographic
100.0%

e. Social, technological, and
political/legal.


Score: 2 / 2

Question 14 (2.00 points)
Which method of entering the global marketplace would be LEAST risky?
Student response:
!ercent
Value
Correct
Response
Student
Response
Answer Choices



a. Exporting
0.0%

b. Licensing
No . . . see pp.66-68.


c. Contract manufacturing


d. A joint venture


e. Direct investment


Score: 0 / 2

Question 15 (2.00 points)
Which of the following is NOT a logistical component of the supply chain?
Student response:
!ercent
Value
Correct
Response
Student
Response
Answer Choices
100.0%

a. Sales forecasting


b. Production scheduling


c. Inventory control


d. Warehousing and materials
handling


e. Order processing


Score: 2 / 2

Question 16 (2.00 points)
Which of the following is TRUE with regard to a buying center?
Student response:
!ercent
Value
Correct
Response
Student
Response
Answer Choices



a. The initiator may not have the
formal authority to also be the
decider or the purchaser.


b. "Deciders and "purchasers
both work in the purchasing
department; the decider is the
purchasers boss.


c. If management has not
appointed a committee to make
a purchase decision, then there
is no buying center for that
particular purchase.
0.0%

d. Marketers sell to a buying
center; they are never members
of one.
No . . . see pp. 115-118.


e. In recent years, formal
guidelines for the formation of a
buying center have been
hammered out by the National
Association of Purchasing
Managers (NAPM), and most
Fortune 500 companies now
follow these guidelines.


Score: 0 / 2

Question 17 (2.00 points)
The performance of services tends to be less uniform than goods. That is, one service provider may perform
the service with a different level of expertise or precision than another service provider. Precision of
performance may also vary for the same service provider depending on the time of day, day of the week,
mood of the provider, or a number of other human factors. This illustrates the __________________ of
services.
Student response:
!ercent
Value
Correct
Response
Student
Response
Answer Choices


a. intangibility
100.0%

b. heterogeneity


c. inseparability


d. perishability


e. portability


Score: 2 / 2

Question 18 (2.00 points)
Owners of a small confectionery have noticed a 20 percent decrease in sales of the companys milk chocolate
candy over the past three years. Theyd like to know why the trend has occurred and how to reverse it. Once
they have identified the problem as a consumer perception that dark chocolate provides health benefits that
milk chocolate does not, the next step in their research project should be to:
Student response:
!ercent
Value
Correct
Response
Student
Response
Answer Choices


a. collect data.


b. recognize the marketing
problem.


c. analyze the marketplace.
100.0%

d. plan the research design.


e. specify the sampling procedure.


Score: 2 / 2

Question 19 (2.00 points)
In the consumer decision-making process, "external information search refers to
Student response:
!ercent
Value
Correct
Response
Student
Response
Answer Choices


a. consumers visiting retail stores
and talking to salespeople in an
effort to gather information
about the best brand to
purchase.


b. consumers looking inside
magazines, books, consumer
guides, and marketer-provided
brochures in an effort to gather
information about the best
brand to purchase.


c. consumers trying to "get inside
the minds of friends and family
members to get a feel for their
opinions as to the best brand to
purchase.
100.0%

d. all of the above.


e. none of the above.


Score: 2 / 2

Question 20 (2.00 points)
Prof. A stops at Sheetz for a cup of coffee to go. As he takes out his debit card to pay for the coffee, he
notices a bin of Cadbury Crme Eggs at the checkout counter and adds two of them to his purchase
transaction. For this consumer in this situation, the Cadbury Eggs are _____________.
Student response:
!ercent
Value
Correct
Response
Student
Response
Answer Choices


a. a generic product
100.0%

b. a convenience product


c. a shopping product


d. a specialty product


e. an MRO supply item


Score: 2 / 2

Question 21 (2.00 points)
Cutter & Buck is a high-end fashion sportswear company. Even though swimwear is a product category that
does not sell year round, its sales remain steady all year because Cutter & Buck sells to wholesale
distributors that stock the product and then ship it to retailers as it is ordered. The wholesale distributors
who buy from C&B are helping to overcome a(n) _____ discrepancy.
Student response:
!ercent
Value
Correct
Response
Student
Response
Answer Choices


a. quantity


b. spatial


c. possession
100.0%

d. temporal


e. assortment


Score: 2 / 2

Question 22 (2.00 points)
Companies that are MOST likely to succeed in the introduction of new products typically:
Student response:
!ercent
Value
Correct
Response
Student
Response
Answer Choices
0.0%

a. make the long-term commitment
needed to support innovation
and new-product development.
True . . . but there's more.
See pp. 172-179.


b. have an obsession with
producing the best product
possible.


c. have a history of carefully
listening to customers.


d. have a vision of what the market
will be like in the future.



e. do all of the above.


Score: 0 / 2

Question 23 (2.00 points)
___________ is/are described by the authors of the textbook as an example of a social influence on
consumer buying decisions.
Student response:
!ercent
Value
Correct
Response
Student
Response
Answer Choices


a. Societal values


b. Stage in the family life cycle


c. Lifestyle
100.0%

d. Reference groups


e. Self-concept


Score: 2 / 2

Question 24 (2.00 points)
Two of the most important pieces of equipment at Cobalt Coal Corporations Westchester mine at Big Sandy,
West Virginia are its Joy continuous miner (for which it paid over $500,000) and its Caterpillar diesel-
powered generator (for which it paid another $500,000). The company would not be able to extract coal
from the McDowell County mountainside without the cuts made by the 40-foot continuous miner, and the
"miner would be useless without the electricity that is provided by the generator. CEO Mike Crowder hopes
that both pieces of equipment will operate reliably for years to come, and on its financial statements Cobalt
"depreciates them rather than "expensing them.


Another item that is essential to the operation of the mine is the heavy-duty, high-voltage electrical cable
that connects the continuous miner to the generator. Because of the tight physical space and limited
visibility inside the mine, the employee who operates the Joy miner (the "miner operator) can easily run
over the cable with the miner, resulting in a crushed or severed cable. Such accidents happen. Cobalt may
have to replace the cable several times per year, at a cost of about $13,000 each time. Because of the
cables short expected life span, Cobalt expenses them rather than depreciating them.


As business products, the authors of your textbook would categorize the continuous miner as __________;
they would categorize the generator as __________; they would categorize the cable as __________.
Student response:
!ercent
Value
Correct
Response
Student
Response
Answer Choices


a. accessory equipment; an
installation; a component part
100.0%

b. an installation; also an
installation; accessory
equipment


c. an installation; accessory
equipment; an MRO item


d. an installation; accessory
equipment; an installation


e. an installation; an instrallation;
a component part


Score: 2 / 2

Question 25 (2.00 points)
One of the four criteria for successful segmentation is "responsiveness. A responsive segment:
Student response:
!ercent
Value
Correct
Response
Student
Response
Answer Choices


a. is one in which at least 50
percent of the consumers are
interested in the product.


b. is large enough to warrant
developing a special marketing
mix.
100.0%

c. responds to the marketing mix
differently than other segments.


d. has known social and
demographic characteristics.


e. can be "reached by marketers
when they attempt to do so.


Score: 2 / 2

Question 26 (2.00 points)
What is the chief advantage of primary data over secondary data?
Student response:
!ercent
Value
Correct
Response
Student
Response
Answer Choices


a. Primary data can usually be
obtained for a lower cost.
100.0%

b. Primary data are likely to be
more relevant to the problem at
hand.


c. Primary data are available for
any interested party to see and
use.


d. Primary data can often be
obtained from a public library.


e. Primary data are immune from
measurement error and
sampling error.


Score: 2 / 2

Question 27 (2.00 points)
Market research has shown that 6- to 17-year-old males, 18- to 24-year-old females, and 18- to 44-year-old
males are the consumer groups who are most likely to watch a televised professional wrestling event or
attend a live match. Wrestling events are regularly broadcast in 120 different countries. Merchandise
licensed by World Wrestling Entertainment (formerly known as the World Wrestling Federation) can be
purchased at JCPenney and Walmart. Marketing decision makers at WWE understand that WWE-themed
action figures, video games, and novelty candy will have greater appeal to fans who are children and
adolescents. Adult fans, on the other hand, are expected to be less responsive to "toy offerings (at least for
themselves) but are still proud to wear clothing that carries the World Wrestling Entertainment (WWE)
theme and trademarks-and the types of WWE-themed clothing that appeals to adult female fans are
different from the types that appeal to adult male fans.


Since the WWE realizes that it needs to use different merchandise and methods to reach its three largest
target markets, it is likely to use
Student response:
!ercent
Value
Correct
Response
Student
Response
Answer Choices


a. market integration


b. concentrated targeting


c. undifferentiated targeting
100.0%

d. multisegment targeting


e. one-to-one marketing


Score: 2 / 2

Question 28 (2.00 points)
Which of the following are NOT mentioned by the authors of your textbook as a factor that plays a role in
shaping a retail stores "atmosphere?
Student response:
!ercent
Value
Correct
Response
Student
Response
Answer Choices


a. Volume and tempo of
background music


b. Degree of density and "clutter
in merchandise display


c. Number of employees who are
visible to a customer at any one
time
100.0%

d. Whether the store is a franchise
or owned by the parent
corporation


e. Fragrances and odors


Score: 2 / 2

Question 29 (2.00 points)
All other things being equal, when brand awareness is high, and the brand is perceived to be of high quality,
and customers of the brand are extremely loyal to it, the brand can be said to have __________.
Student response:
!ercent
Value
Correct
Response
Student
Response
Answer Choices
100.0%

a. high brand equity


b. a monopoly


c. brand security


d. synonymity


e. economies of scale


Score: 2 / 2

Question 30 (2.00 points)
Relationship marketing programs for services have the greatest potential for sustaining long-term
relationships when they are based on ___________ bonds.
Student response:
!ercent
Value
Correct
Response
Student
Response
Answer Choices


a. municipal, debenture, and zero-
coupon


b. bail, inflation-linked, and
treasury
100.0%

c. financial, social, and structural


d. convertible, social, and
structural


e. retractable, financial, and
structural


Score: 2 / 2

Question 31 (2.00 points)
What does the term "product positioning refer to?
Student response:
!ercent
Value
Correct
Response
Student
Response
Answer Choices


a. Efforts by marketers to have
their brands used by high-
visibility characters in movies
and TV shows


b. Efforts by marketers to secure
the best shelf locations in major
chain supermarkets and
drugstores


c. Geographic segmentation, often
within major metropolitan areas
100.0%

d. The place a product occupies in
consumers minds-in
comparison with competitors
products-based on important
features


e. The ranking of a product in
Interbrands (a global consulting
group) annual "100 Best Global
Brands. For example, Coca-
Cola is ranked #1 in the 2010
"100 Best Global Brands and
Hewlett-Packard is ranked #10)


Score: 2 / 2

Question 32 (2.00 points)
Dell has no desktop PCs, notebook PCs, netbooks, or tablet PCs in inventory waiting for someone to order
them. Dell is considered a pioneer in the "configure to order approach to manufacturing; it does not build a
computer until the computer has been ordered and the customer has specified the exact combination of
features for that computer. The authors of your text call this:
Student response:
!ercent
Value
Correct
Response
Student
Response
Answer Choices


a. push distribution.


b. niche marketing.


c. psychographic segmentation.
100.0%

d. mass customization.


e. a production orientation.


Score: 2 / 2

Question 33 (2.00 points)
In a typical channel of distribution for a low-cost, frequently-purchased consumer product such as a
magazine or package of candy, a wholesaler would primarily sell to and be concerned about the
___________, and a manufacturer would primarily sell to and be concerned about the ___________.
Student response:
!ercent
Value
Correct
Response
Student
Response
Answer Choices


a. consumer; retailer


b. manufacturer; retailer
100.0%

c. retailer; wholesaler


d. broker; manufacturers
representative


e. retailer; consumer


Score: 2 / 2

Question 34 (2.00 points)
The population of Madisonville, Kentucky, is approximately 19,000. The population of Danville, Kentucky is
between 15,000 and 16,000. The population of Somerset, Kentucky is approximately 12,000. Each of these
towns has one Ford dealer: Hudson Ford-Nissan in Madisonville, Stuart Powell Ford-Lincoln in Danville, and
Alton Blakley Ford-Honda-Mazda in Somerset. From these statistics, we can conclude that Ford Motor
Company engages in
Student response:
!ercent
Value
Correct
Response
Student
Response
Answer Choices


a. intensive distribution.


b. deficient distribution.


c. selective distribution.
100.0%

d. exclusive distribution.


e. disintermediation.


Score: 2 / 2

Question 35 (2.00 points)
A new medication to eliminate fungal growth on toenails may have a slow rate of adoption because it is not
a highly-visible item in peoples homes. Also, the item is not likely to be discussed often among friends and
coworkers. This is an example of how the characteristic of ______________ can work to slow the adoption
process.
Student response:
!ercent
Value
Correct
Response
Student
Response
Answer Choices


a. relative advantage
100.0%

b. observability


c. trialability


d. complexity


e. compatibility


Score: 2 / 2

Question 36 (2.00 points)
A(n) _________ is a form of personal interviewing that uses a group of seven to ten people who have been
recruited because they have certain desired consumer characteristics.
Student response:
!ercent
Value
Correct
Response
Student
Response
Answer Choices


a. passive people meter


b. executive panel
100.0%

c. focus group


d. dialectical cohort


e. SME (subject-matter expert)
debriefing


Score: 2 / 2

Question 37 (2.00 points)
Coca-Cola, Planters, and Purina One are _____________ brands; Big K (sold by Kroger), Archer Farms
(sold by Target), and Ol Roy (sold by Walmart) are ____________ brands, also sometimes called
___________ brands.
Student response:
!ercent
Value
Correct
Response
Student
Response
Answer Choices


a. generic; knockoff; national


b. private; gray-market; generic


c. house; national; private
100.0%

d. national; private; store


e. step-down; generic; gray-
market


Score: 2 / 2

Question 38 (2.00 points)
The authors of your text use the "gap model to illustrate discrepancies that can cause problems in service
delivery. These same discrepancies can influence customer evaluations of service quality in a negative way.
Which of the following IS one of the gaps in the gap model?
Student response:
!ercent
Value
Correct
Response
Student
Response
Answer Choices


a. The gap between the level of
service that is available for a
bargain price and the level of
service that is available for a
premium price


b. The gap between the level of
service that is provided by the
#1 rated company in the
industry and the level of service
that is provided by the second
best


c. The gap between the level of
service that is provided by a
union employee and the level of
service that is provided by a
nonunion employee


d. The gap between the quality of
service employees are trained to
provide and the quality of
service they actually provide
100.0%

e. The gap between what the
company provides and what the
customer is told it provides


Score: 2 / 2

Question 39 (2.00 points)
Which of the following statements about how adopters participate in the diffusion process is FALSE?
Student response:
!ercent
Value
Correct
Response
Student
Response
Answer Choices


a. Some product categories are
never adopted by the entire
population.
100.0%

b. The late majority are the last
consumers to adopt a new
product.


c. Early adopters are more likely
than innovators to be opinion
leaders.


d. Skepticism is a dominant
characteristic of members of the
late majority.


e. Innovators are the first to
purchase a new product.


Score: 2 / 2

Question 40 (2.00 points)
Friedas, Inc. is an Orange County, California-based specialty produce company that supplies supermarkets
and restaurants (including Kroger, Walmart Supercenter, and Acme Market locations in West Virginia) with
more than 600 varieties of fruits, vegetables, and gourmet items. The companys founder, Frieda Caplan,
made history in the early sixties when she began importing the Chinese gooseberry (which she renamed
"kiwifruit) into the U.S. Friedas current corporate slogan is "We change the way American eats. Indeed,
the company can make this claim honestly. In addition to kiwifruit, Friedas has introduced more than 180
exotic and specialty items to supermarkets and distributors throughout the U.S., including shallots,
cherimoyas, Donut peaches, sugar snap peas, and Habanero peppers. Friedas, Inc. is a __________.
Student response:
!ercent
Value
Correct
Response
Student
Response
Answer Choices
100.0%

a. wholesaler


b. 3PL ("third party logistics)
provider


c. producer


d. category killer


e. franchisor


Score: 2 / 2

Question 41 (2.00 points)
A marketing executive who believes in the "total CSR approach to social responsibility will adopt the
perspective of the "pyramid of corporate social responsibility, which contains all of the following
components EXCEPT:
Student response:
!ercent
Value
Correct
Response
Student
Response
Answer Choices


a. economic responsibility.


b. legal responsibility.



c. cultural responsibility.


d. ethical responsibility.
0.0%

e. philanthropic responsibility.
See pp. 35-37.


Score: 0 / 2

Question 42 (2.00 points)
Retailers control the six Ps of the retailing mix. The six Ps include the four Ps of the marketing mix PLUS:
Student response:
!ercent
Value
Correct
Response
Student
Response
Answer Choices


a. procurement and promise.


b. processing and perception.


c. palletization and packaging.
100.0%

d. personnel and presentation.


e. patronage and participation.


Score: 2 / 2

Question 43 (2.00 points)
Which of the following is CORRECT regarding the product life cycle?
Student response:
!ercent
Value
Correct
Response
Student
Response
Answer Choices


a. A long-run drop in sales usually
signals the beginning of the
maturity stage.


b. Even though the product life
cycle concept appears in most
marketing textbooks, it is of
little practical value to marketing
managers and planners.


c. Production and promotion costs
are particularly high during the
growth stage, and the company
or companies producing the
product experience(s) a loss
rather than a profit.
0.0%

d. Normally, the longest stage of
the product life cycle is the
introductory stage.
No . . . it's the maturity
stage. See pp. 181-184.



e. The degree to which marketers
have to educate the public in
order to make the product
known can have an effect on the
length of the introductory stage.


Score: 0 / 2

Question 44 (2.00 points)
Which of the following does NOT describe the business market, as compared to the consumer market?
Student response:
!ercent
Value
Correct
Response
Student
Response
Answer Choices
100.0%

a. A business buyer is much less
likely to try to negotiate a price
than is a consumer.


b. Business customers tend to be
more geographically
concentrated, and customers in
consumer markets tend to be
more geographically dispersed.


c. Business markets are served by
a more direct channel of
distribution than are consumer
markets.


d. Purchasing by businesses is a
more formal process than
purchasing by consumers.


e. Business marketers tend to have
far fewer customers than
consumer marketers.


Score: 2 / 2

Question 45 (2.00 points)
With regard to marketing by nonprofit organizations, with which of the following statements would the
authors of your text likely AGREE?
Student response:
!ercent
Value
Correct
Response
Student
Response
Answer Choices


a. The marketing mix for nonprofit
organizations consists of 3 Ps:
product, place (distribution), and
promotion.


b. Nonprofit organizations usually
market products or services that
are by definition "very high
involvement for the recipients
and users of those products or
services.


c. The benefits of nonprofit
organizations product offerings
usually are easier to
communicate than those of for-
profit marketers.
0.0%

d. All of the above.
No . . . at least two of "a"
through "c" above are bogus.
See pp. 196-199.



e. None of the above.


Score: 0 / 2

Question 46 (2.00 points)
The SECOND task of developing a retail strategy is to:
Student response:
!ercent
Value
Correct
Response
Student
Response
Answer Choices


a. define the target market.
100.0%

b. choose the retail mix.


c. hire the employees.


d. design the stores website.


e. hire an advertising agency.


Score: 2 / 2

Question 47 (2.00 points)
All of the following are elements of the marketing plan EXCEPT:
Student response:
!ercent
Value
Correct
Response
Student
Response
Answer Choices


a. a mission statement.


b. the situation analysis.


c. the target market strategy.


d. the marketing mix.
100.0%

e. a portfolio analysis.


Score: 2 / 2

Question 48 (2.00 points)
Using the BCG (Boston Consulting Group) matrix, how would Unilever categorize itself?
Student response:
!ercent
Value
Correct
Response
Student
Response
Answer Choices


a. As a dog


b. As a star
0.0%

c. As a cash cow
A large, diversified
corporation such as Unilever
WOULD NOT categorize its
overall organization using
the BCG matrix; it would
categorize each of its SBUs
(e.g., foods, home care,
personal care), or its product
lines (e.g., condiments and
dressings, laundry products,
bath products), or its
individual brands (e.g., Wish-
Bone, Surf, Dove, Axe). See
pp. 23-24.


d. As a problem child



e. As none of the above.


Score: 0 / 2

Question 49 (2.00 points)
Even though they do not have authorization from manufacturers, distributors will sometimes divert products
from low-price markets to sell them in high-price markets. For example, a consumer in the U.S. who is
shopping for a new Yamaha grand piano may find one for $27,500 and another very similar one for $14,000.
The difference is that the $27,500 piano was built for the U.S. market; the $14,000 piano was built for the
southeast Asia market and was intended to be sold at a list price of $8,500 in that market. Through
unauthorized diversion to the U.S., some distributor (which has not been authorized by the Yamaha
Corporation) makes additional money, even though the consumer pays less. (The consumer may actually be
ripped off, though, because the wooden parts of the piano have been carefully selected and treated for
optimal performance in the humidity of southeast Asia; those parts may crack or wear out prematurely from
becoming excessively dry in some areas of the U.S.) This scenario is an example of
Student response:
!ercent
Value
Correct
Response
Student
Response
Answer Choices
0.0%

a. dumping.
No . . . in this scenario,
"dumping" the less-
expensive piano in the United
States would involve pricing
the piano at less than $8,500
(its price in its intended
market, southeast Asia). See
p. 72, pp. 213-214.


b. price fixing.


c. countertrade.



d. the gray market.


e. price escalation.


Score: 0 / 2

Question 50 (2.00 points)
The ____________ at John Lewis, Britains largest department store chain, selects the merchandise for his
or her department and is often responsible for its promotion and for managing the personnel in that
department.
Student response:
!ercent
Value
Correct
Response
Student
Response
Answer Choices
100.0%

a. buyer


b. franchisee


c. consignment agent


d. broker/purveyor


e. receiving clerk


Score: 2 / 2

Você também pode gostar